You are on page 1of 54
Unit Chemical Kinetics ES ‘Ate studying this Unit, you will be able to: define the average and instantaneous rate of @ reaction; ‘express the rato of reaction in terms of change in concentration of eter ofthe reactants or products with time; 2Na() + 3N@) GNa(t) + Fe,05(s) —> 3Na,O(s) + 2Fe (0) 2. Very slow reactions (Type 2). This type comprises reactions which oovur tt very slow rate, These reactions may require montis# even years together for their completion. For exams © Rusting is a ‘slow’ reaction; you hardly see change looking at it ! © The weathering of rocks is an extremely" slow reaction. * The fermentation of sugar to alcohol is quilt slow but you can see the carbon dios bubbles forming in the ‘froth’ in a labo experiment or beer making in industry! yet’ "08 Of such reactions are hardly 4 Physical importance, Scanned with CamScanner rately slow reactit ode? vofers to reactions in between the very e hi ond very slow (type 2) reactions, These (OP eed at moderate speed which can be easily 1 fot Pre trcion of eae sugar ancl hydrolysis of Invern examples ofthis typo of renetions, 4,0 +H,0() —> CoH20g + CoH, .0, oy Glucose Fructose + nH,O—> nC yg y,0,, fh OF A CHEMICAL REACTION rate of a chemical reaction is the speed or aten a reaction takes place. te of chemical reaction may be ‘aany ofthe following ways: sie he rate of deerease in concentration of any ofthe reactant, OF | one iy Te rate of increase in concentration of any the product. ; oes example, for a hypothetical reaction, R—>P et us assume that the volume of the system sins constant. TI[R}, and [P] are the concentrations of R and P sepcively at time f and (RJ, and (PJ, are their sonentrations at time f then, At= tt A(R] = (R), - FR); AP) = Pl, - Ph Rate of disappearance of R __ Decrease in concentration of R ‘Time taken At Rate of appearance of P _ Increase in concentration of P _ , A(PI ‘Time taken bt The expressions for rate in terms of reactants ‘ary a negative sign. A(R] SIGNIFICANCE OF NEGATIVE SIGN = negative sign in first expression does not ut the rate is negative, rather it indicates the Rn es toed sare brackets inthe above expressions are used ‘molar concentration. decrease in concentration of the reactant. As we knov’ that with passage of time the concentrations of reactants decrease, therefore, the change iP concentration, A{le] = (Final cone. ~ Initial cone.) will have a negativo value. Since the rate of a reaction is 8 positive quantity, thua, to get a positive rate we PUES ve sign in the rate expression. ‘Thus, Rate of reaction = Rate of disappearance of R = Rate of appearance of P = PL > The qualitative rate of a reaction is expressed in terms of parameters like physical disappearance of reactants, colour change, effervescence, ete. Such observations provide arbitrary conclusions about the rate of a reaction. ‘= The quantitative rate of a chemical reaction provides us information about the rate data and the accurate rate of a reaction. MATHEMATICAL EXPRESSION FOR RATES OF REACTION 1. Reactions Involving Same Stoichiometric Coefficients of all the Reactants and Products. Let us consider the gaseous reaction between nitrogen dioxide and carbon monoxide. NO,(g) + CO) ——> CO,(@) + NO) In this case, as the reactants and products appear in same stoichiometric proportions, therefore, the rate of the reaction may be expressed in terms of rate of disappearance of nitrogen dioxide or carbon monoxide or alternatively, by the rate of appearance of carbon dioxide or nitric oxide whichever may be convenient. ‘Thus, rate of reaction is mathematically given. as, AfNO] At A(NO,} __ AICO] _ , A(COs] at at At Nitrogen dioxide is a deep reddish brown gas, and therefore the rate of this reaction can be followed more easily by measuring the change in the intensity of colour in the gaseous mixture in a given interval of time. Scanned with CamScanner lar manner the rate expres eactions ean be written Fora reaction, Hg(!) + Cl) —— HgCl,(s) Rate of reaction =— A, _ Cla], , tela] wae aw 2. Reactions Involving Different Stoi- chiometric Coefficients of Reactants and Products, Let us take an example of simple renction, H,@) + 1,@——> 2H) Now, in the present case, as the reacta: (Hi, and I,) and products (HI) have different stoichiometric coefficients, this means that for overy mole of H, or I, reacting we get two moles of HI. Thus, the rate of formation of HI will be twice the rate of disappearance or Hy or Ip, In such cases, we divide the rate of change of concentration by the stoichiometric coefficient of reactant or product involved in the reaction, Thus, we have . _ A(Hy] Aly] _ 1 ACH Rate of reaction =- 7 =-— = 9 ay Similarly, for the reaction, Br (ag) + Br,” (aq) + 6H* (aq) —> 3Br,(aq) + 3H,0() Rate of reaction =~ 2-08#7}__ B05] at _ 1 4040] “3 at ao Remenber ‘The rate expressions are divided by coefficients to @ avoid difference in rates and (ii) to get unique rate of reaction. In general, for a hypothetical reaction, aA + 6B > 0 + aD AG] a UNITS OF RATE OF REACTION The units of rate of reaction are concentration time“. As concentration of substance is expressed in mol L-! and the time is expressed in seconds or minutes or hours the units for reaction rate, therefore, are Er, 5 moles litre"! sec! or moles litre- litre hr“ min 8 For gaseous reactions, whose con, my given in torms of partial pressures thea, Mati Fonction will bo atin min“ or atm goes ys, at ty PB Remember For a gaseous reaction at consteny ~~ concentration is direetly proportions} mea pressure of a spocies and hence, rats”. pays expressed as the rate of change in hee a the reactant or the product, Prete, ‘Tho partial pressure of any gaseous com, bo calculated by any of the following meth, ( From ideal gas equation, pV = apy or When V, 7, etc., are given, i) When mole fraction is given then by expression sing P= Pra * mole fraction of 08 in question Inter conversion of units. The partalya can also be converted into units of concentra, moles per litre, by using the following equate he pV=nRT or pay RT=ekr or e=k ® Solved Problems Based on) | Rate Equations =}. | | one A gaseous hypothetical chemical reccis 2A === 4B + C is carried out in a closed esi. concentration of B is found to increase by 5% 10% mal it 10 seconds. Calculate (0 Rate of appearance of B (ii) Rate of reaction and (iii) Rate of disappearance of A. Solution. ( Rate of appearance of B _ Increase in cone. of B 5x 10+ mol Ls =t x5 x 10-4 = 1.25 x 10+ mol L'# Scanned with CamScanner Fr i Wi 1 x Rate of disappearance of B a dxsx10+ = mal 17 et = 2.5 x 10+ mol L*t st, The rate of formation of nite goin: reaction i 2.6% 102 mol Lit gin (10) rade) + 50d) —> ANO(e) + 611,0(4) sind the rate of disappearance of oxygen. aan he rate of above reaction in terms of oxygen and solu yo# 1 (02) _, 2 [NO] pat aw rato of disappearance of oxygen 5 = G (rate of appearance of NO) 5 3x 816 10° = 4.5 x 10° mol Lots, po For the reaction, 2N,0fe) <== 4NOfe) + Ox) rration of NO, increases by 3.0% 10 mol E-"in igthe concent A il econds then what is the rate of reaction? Solution. Rate of reaction og} (NOn] _ 1, 3x10 o" 4 bt 4 6 21.25 x 10+ mol Ls. The oxidation of iodide ion by peroxydisulphate ion is given as: sf +8,0-—— I, + 280, 2 @ 22000) = 1.5% 107 mol Ls" fora given AUT] for the same interval than what is the value of interval? (i) Also calculate the rate of formation of SO;* for the same interval. Solution, According to the given equation: 1a} __ Af8:057) At at 1 a(80,"7] a2 (At F soe) 23x 1.5% 109=4.5%1 0-3 mol L7s* «i aol AI8,0, ca] are =1.5*109x2=3,0x 10% mol Ls im — If the rate of formation of oxygen gas is ae at be rate of a reaction is always expressed in terms 43 0,= 8 =1.5 mot mrt (0 rate of disappearance of NO 1 AINsOg) _ A103) i aemeem a = 2x 1.5=3.0 mol mm (Gd rato of appearance of NO,, 1 AINO,] _ 403) aa =4X 1.5 = 6.0 mol ma. 1. Express the rates of the following reactions in terms of the concentrations of reactants and products: () PC, —> PCL, + Cl (i) Ny + 3H, —> 2NH, (i) 2NO, + Fp —+ 2NO.F (jv) 2N,0, —> 4NO, + 05 2, For the following gaseous reaction, 2NO,—— 2NO + 0, () Write the expression for the rate ofthe reaction. (i Tethe rate of decrease of concentration of NOs is 14.0 x 10-1" «1, what are corresponding rates of increase in NO and O, concentrations? 4. The reaction, 2N,0,(@) == 4NO,() + 0218), takes place in a closed flask. It is found that Concentration of NO, increases by 20% 10° mol L* fab seconds. Calculate the rate of the reaction and the rate of change of concentration of N,O;, on, 21° + Cla(g) —> I, + 2CF if the ation of was 0.80 mollitre and cone. 0.68 molflitre, calculate the fT" and rate of appearance 4, For a reacti initial concent after 20 minutes was rate of disappearance o} ofl, Scanned with CamScanner 200°C, N,0,(¢) — N,0,@) + Fox, if the initial pressure is 114 mm and after 25 minutes of reaction the total pressure of gaseous mixture is 133 mm, calculate the average rate of reaction in (a) atmosphere min"! and (6) mol Z~ts-". 6. The decomposition of N,O, in CCl, at 918 K has been studied by monitoring the concentration of 1,0, in the solution. Initially the concentration of NO, is 2.33 mol L*! and after 184 minutes, it is reduced to 2.08 mol L-*. The reaction takes place according to the equation 2.N,0, (¢) — 4 NO, (@) +0, @) Calculate the average rate of this reaction in terms of hours, minutes and seconds. What is the rate of production of NO, during this period? (NCERT Textbook Solved Example 4.2) 2 () 4.0% 10-9 mol Lt (ii) 2« 10° mol I-15". 8. 110% mol Et st; 2% 10-9 mol Lt &, 4. Rate of disappearance of I- = 0.006 mol L! mint Rate of appearance of I, = 0.008 mol Lt min-!. 5. Average rate (a) 2x 10% atm min! 6. Average rate () 4.07 * 10% mol Lhe (ii) 1.18% 10° mol I>! § A WORK OF LAXMI PUBLICATIONS (P) LTD. @ AVERAGE RATE AND INSTANTANEOUS RATE The average rate of a reaction is defined as the rate of change of concentration per unit time. It is calculated by dividing the total change in concentration. of any one of the reactant or product by the total time taken to do so. For a reaction. Rk — P (product) (6) 8.59 x 107 mol L-! st, (reactant 7 : ; ant rn te SS ----------------- erage ra 5. For the decomposition of dinitrogen pentoxide at ‘ ; On in gi n ie, ‘The instantaneous rate of a reactio, as the decrease in concentration of any minds, reactants or increase in concentration of any A product at a particular instant of time jot, temperature. aR Mathematically, the instantaneous rate May, written as, at aR) apy Tiga = or Ge or + TPL Here, dtis the infinitesimally small tne, and de is the change in concentration of any species in time interval dt. th Do You Rese? “In| mall anf | Ax approaches zero, the ratio (average rate) mat calculus, when At becoi ferent replaced by the derivative & which means rate change of x with time ‘t’ It may be put as Jim St — de, a at > ae AIR] To Sum up, the expression or ae dx diR} dP b Bor ches 0 ecomes Sor TE) oy PPI as at approw ie, the average rate approaches the instantaneots? as At becomes smaller and approaches zer0, ies hf At}y 40” dt For example, for the gaseous reaction, 2N,0, —— 4NO, + 0, The Instantaneous rate = IN:0,} _ 1 aN04) _ ‘inst t dt dt Scanned with CamScanner ipeesinlcl eoelfisionta axe aT rape rate ofa reaction. These coofiionts are ante not to be used while expressing the rato of over veldisappearance ofthe reaction, sore awe even exile, torpor disappearance of NO, dtN205] at = 2 rate of reaction similarly, alNO,1 rate of appearance of NO, = N03) Taina, Of N2Og x2 rate of reaction. pxPERIMENTAL, DETERMINATION OF RgACTION RATE herate ofa reaction is determined by measuring peconentraion of any of the reactants or products ecomifiite interval of time. For this purpose any fer le property such as change in volume, pressure, asoe index, pH, eletrical conductivity or thermal tefl, et which is related to the concentration conteces participating in the reaction is selected. The lange in this property is measured as a function of (ne. The common practice to find the change in line. gtration is to withdraw a small amount of reaction mixture (2m? to5 cm‘) at different intervals "Htime, The sample is then placed in the freezing of ature ofice and sodium chloride. Its called freezing ofreaction. The concentration at the given interval is then determined by a suitable method. On the basis of netic data a graph is plotted between concentration and time (Fig. 4.1) and the rate is calculated. Concentration (molesfitre) Time (seconds) +} “Fig. 4.1. Determination of rate of a reaction. respect to the time dotermined. The differen sponding to these points is calculated. This difference of concentration is then dividi : ion is then divided by the time interval between the selected equi ci 181 CALCULATION 01 FAN REAGHAS \VERAGE RATE OF THE, Suppose Fig. 4.2 is obtained by plotting concen tration of renctatte (ht enctants (Fig. 42a) and concentration of Drodtets (Fi. 1.20) vera tine se On the axis, of time, two points are taken with at which the average rate is to be in concentrations corre- tant points. Accordingly in the Fig. 4.2(a) and Fig. 4.2(b), the average rate can be calculated by measuring the change in concentration in the time interval from f to ty (Rly Concentration of reactants —> wat Time —> Fig. 4.2 (0) tous SEL. lope Concentration of products —> Fig. 4.2 (b) ‘Thus, average rate at time ‘t, = 7fRal=(Rl = ok Similarly, the instant rate of a reaction at time “P18. Tigge = Slope of tangent. Scanned with CamScanner 4 Do You Kuaw? The average rate at a time can also be calculated by taking two equidistant points to time ¢. The difference in concentrations corresponding to these points is calculated. Hence, ARI, 24 MP) rye MEL op = + SE For instant rate at time ‘t, or straight line is drawn to cut the curve at a point P. A tangent is drawn to ‘curve at point P. Hence, Ton = slope of tangent Solved Problems Based on Average Rate of a Reacti n NCERT INTEXT QUESTIONS: Far the reaction R— P, the concentration of a reactant changes from 0.03 M to 0.02 M in 25 minutes. Calculate the average rate of reaction using units of time both in minutes and seconds. Solution. The average rate of the reaction will be _ (Rb -1Rh fh ‘Now, (RJ, = 0.02 M, [R], = 0.080 M, A¢ = 25 min Average rate 8 001) 25 54104 M mint = 6.66 x 10° Ms, In a reaction, 24 -+ Products, the con- centration of A decreases from 0.5 mol L-! to 0.4 mol I-! in 10 minutes. Calculate the rate during this interval? Solution, Average rate of reaction 14A__1/[Ab-[A} 2a 2 ag Now, [A], = 04M, [4], =0.5 M, at = 10 min ++ Average rate=— 104-05 __ 1 (03) 27 10 2 107 M min, of kinetic data, obtained for the reaction, (CH,COOH (aq) + CH,OH (ag) —E*_, CH,COOCH, (ae) + 1,0 7220 30 40 60 60 70 e030 ih" From the plot calculate: (0 average rate of disappearance of acetic acid ani (i) average rate of appearance of methyl acetate, (ii Instantaneous rate at 55 seconds. 2, From the concentrations of CHCl (butyl chlor) at different times given below, calculate the average rate of the reaction: C,H,Cl+H,0 —> C,H,OH + HCl during different intervals of time. us o | so | 100 | 150 |(C,H,Cl)/ | 0.100 | 0.0905] 0.0820 [0.0741 mol Lt tls 300 | 400 | 700 | 800 (CHCl) | 0.0549] 0.0439] 0.0210 0.017 mol L-! (NCERT Textbook Solved Example Scanned with CamScanner of diss. of acetic acid = 0.0186 mol I-11 4. rate arate (id Tot pc veragerateis calculated by dividing diference g. TheGT over different time intervals by difference in incone. over a ee Accordingly, average rate for the 8 time intervals are! imgp x 104 1.70 * 104 1.58 x 10%; 1.40 x 10%; 99104 11 104 1.04% 104 04% 10% mol Et et respective: (gion on RERTORICANONS DT Fhe concept of mechanical speed cannot be used to ‘ihe rate of a chemical reaction. This is because mem be constant throughout the reaction. Tnactual, the rate ofa reaction always varies with tine, Asthe molar concentration of reactants fall with tine the rate ofa reaction also decreases. Itis therefore vee appropriate to express the rate as instantaneous rate. FACTORS INFLUENCING RATE OF A REACTION ‘The rate of reactions is influenced by the following factors: () Concentration of the reactants, (ii) Temperature of the reactants, (iil) Noture of the reacting substances, (iv) Presence of catalyst, and (©) Exposure to radiations. DEPENDENCE OF RATE ON q CONCENTRATION When a chemical reaction occurs, the reactants ‘hange over to products. It is observed that with the Passage of time the concentrations of reactants decrease lethose of products increase. It is graphically shown ing. 43, 183 i we 3 5 | 8 Reactions mo Fig. 4.8. Time dependence of the concentrations of Feactants and products in a reaction. Now, if we assume that other factors are constant then the rate of a chemical reaction decreases with the decrease in concentration of the reactants. Conversely, rates generally increase when reactant concentrations increase. For example, we find that a piece of wood burns at a much faster rate in oxygen than in air. It is because of higher concentration of O, in the former. ‘The representation of the rate of reaction in terms of concentration of reactants is known as rate law. It is also called rate equation. 2 eemenber A qualitative relationship between the rates of reactions and the concentration of the reacting species is known as “Law of Mass Action”. For a hypothetical reaction; A+B— Products according to law of mass action Rate of reaction, r« (AJ(B] = MIAIB) where (A] and [B] are the molar concentrations of the reactants A and B respectively and k is a constant of proportionality. Rate Expression and Rate Constant Consider a general reaction aA +bB——> cC + dD (Here, a, b, cand dare stoichiometric coefficients) ‘The rate equation for the reaction is Rate « (A}* [BP ) =k (A (BP oe | - SB =n car BP -@) Scanned with CamScanner 184 (xand y may or may not be equal to stoichlometric coefficients a and b) The equation (1) which relates the rate of a reaction to concentration of reactants is called rate law or rate expression. Thus, rate law is the expression in which reaction rate is given in terms of molar concentration of reactants with each term raised to some power, which may or may not be same as the stoichlometric coefficient of the reacting species in a balanced chemical equation. ‘The equation (2) is known as differential rate equation. ‘The constant h is a proportionality constant and is known as rate constant or velocity constant or specific reaction rate. If the concentration of all reacting species is taken as unity then, rate =k [+ [A] = (B] =1 mol Ly Hence, rate constant is defined as the rate of reaction when the concentration of each reactant is taken as unity. it is for this reason that the rate constant is also called specific reaction rate. Deriving the Rate Expression from Data There are many different ways and methods to determine the rate expression for a reaction. One experimental method that is used to determine rate laws of chemical reactions is the method of initial reaction rate, INITIAL RATE METHOD The initial rate of a reaction is the instantaneous rate determined just after the reaction begins (t = 0). In this method the basic idea is to determine the instantaneous rate before the initial concentrations of reactants have changed significantly. Several experiments are carried out using different initial concentrations, and the initial rate is determined for each run. The results are then compared to see how the initial rate depends on the initial concentrations, On the basis of kinetic data the form of rate law is determined, Let us illustrate the method of initial rates using the following equation: NH,* (a9) + NO,> (aq) —s N,(g) + H,0(0 The general form of the rate law for this reaction is, a[Nuj] dt ving om exPerimental studies, the following table siving initial rates is formulated from. three experiments involving different concentration of reactants, rate =h(NH,)" (No, Experiment | Initial (NH,‘] | Initial (NO5 (mol L-)| (mol I>) 1 0.100 M 0.0050 M 2 0.100 M 0.010 M_ 200M 0.010 3 o. ML saosin The initial concentration data can hy determine the values of the rate exponents ne “ The strategy is to find two experiments in yt one of the initial concentrations is changing pt} other is kept constant. Accordingly, we can ts dependence of the rate on the change jy "tts concentration. iti In the given kinetic data table, in experin, and 2, the initial concentration of NH,* remei same but the initial concentration of NO,~ doubt observed initial rate also doubles. Using oT expression, we get on rates, 1 = [0.1]” (0.005}" rates 2 =F [0.1]” (0.010]" Dividing eqn. (ii) with eqn. () Tat@op2 _ k[1.0]" (0.010]" [eae " rata 1 (1.0]” (0.005) ag] 2.70107 2) o 135x107 2=(2)". Hence n= or Ina similar manner the results of experineg 2and 3 yield the ratio. 0x 107 _ [0.21 [0.017 _ r (0.1)" (0.017 or 2=2", Hence m=1 Conclusion * The value of m=1andn=1 + The rate expression of the reaction i) therefore rate = k[NH,*] [NO,"] | * The differential form of this rate expressi® is given as dR] dt a Remember ‘The rate law for any reaction cannot be prelit! | by merely looking at the balanced chemical ent fe, theoretically but must be detersitt | experimentally, (NH,*] (NO,"] Scanned with CamScanner d Problems Based o = ial Rate Method ‘The kinetic data for a reacti oma, is a reaction, 24 + p, = Faperiment | [AI (mol L*) | (8) (mot) ] Tait Rage Dorive Rate 222k (ectoen ort Rate lw exrenion apd calculate the value 1 0.50 0.50 1.6% 10-4 a 0.50 100 | aexi04 3 1.00 100 | 32x104 Write the rate law expression. 1. Rate =k [NOP (ly. 2 Rate =k (A) BP; k= 3.0% 107 mot? L? mint. AWORI ‘K OF LAXMI PUBLICATIONS (P) LTD. EE] solution. The rate lav for the reaction is, r= MAB, Calculation of x * ‘The rate expression of experiment 3 is divided with shat of experiment 2 as the concentration of B is kept fonstant CHARACTERISTICS OF RATE CONSTANT 32x10% _ kI(LoFILop ( The value of rate constant gives an idea about * gax10+ k(OsytLoY the speed of a reaction, i.e., greater is the value of rate constant faster is the reaction. In 1= 2 y =@ simple words it is a measure of rate of reac- o 05. tion, oe (= 2, Hence x=0 be @=n (i Each reaction has a definite value of the rate Similarly, the value of y is calculated by dividing rate expression of experiment 2 with that of experiment 1, 32x10 _ kLO5P{L00P constant at a particular temperature. (ii) The value of rate constant depends on the 16x104 [05 P(05P r= (LOY. @ (® 2 Hence, yal Rate law expression, rate = k{A]° [B,]". 1. For the reaction, 2NO + Cl, ——> 2NOCI, the following kinetic data was obtained. Ex. | [NO] [Cl Rate (mol L-), | (mol L!) | (mol ts) 1 0.380 0.380 | 5.0x 10% 2 0.760 0.760 | 4.0% 10% 3, 0.380 0.760 | 1.0% 102 Derive the Rate law expression. 2. The rate data obtained for a hypothetical reaction, A+B— 26, is: temperature i.e., changes with change in temperature (sections 4.11 and 4.13). (iv) The value of rate constant is independent of the concentration of reacting species. (0) The units of rate constant depend on the order of reaction (section 4.7). Remenber + Therate expressions on the basis oflaw of mass action are written on the basis of the balanced chemical equations. ‘+ The concentration dependence of the reaction rate is. determined experimentally and is given by the expression called rate law expression. DIFFERENCE BETWEEN RATE OF REACTION AND RATE CONSTANT ‘The important differences between the rate of the reaction and rate constant of the reaction are being given as follows in tabular form. Scanned with CamScanner eS Ce, rn ee ———L!LULU | Rate of Reaction int of propor is constai 1. Tis constant aie tionality in expression. 1. It is the speed at which| the reactants are conv- erted into the products at any moment of time. 2.It depends upon the| concentration of renetant species at that moment of| time. rate of| specific concen 2.1t refers to the reaction at the int when pelfon of every nencting species is unity, 3, Itisconstant ané depend on the prow the renetion. does not 3.1t generally decreases| mee with the progress of| reaction. RITY OF A MOLECULA) REACTION ‘a reaction called Another important property of y Jaw expression molecularity helps in deriving the rate ‘and mechanism of the reaction. ‘According to Collision Theory a chemical reaction occurs due to collisions between the particles of the reactants. The number of reacting species (atoms, ions or molecules) taking part in an elementary reaction, which must collide simultaneously in order to bring ‘about the chemical reaction is called molecularity of the reaction. The molecularity of the reaction can be 1, 2or 3. For example, ( The decomposition of ammonium nitrite is a unimolecular reaction. NH,NO, > N, + 2H,O _ (Gi) The dissociation of HI is a bimolecular reaction. 2H) —> Hg) + 1,(@) (iii) In the same way, the reaction be i » etwer and O, is a trimolecular reaction because it cae collisions among three species es 2NO +0, 2No, ____ Inalllthe above chemical reacti is simply the sum of molecules of the di as represented by the balanced chemic reactions are known as elementary ions molecularity ifferent reactants ‘al equation. Such i‘ tm reactions, ‘or most of the reactions, the m i , the molecularit not exeeed three. It is because the chances of areca neous collisions between three or more nye nt™ are Particles arg OU, Dotpuu a aa 1 Fe of molecularity not exceed; ‘he lowe ee basisof daily experant fa conclu ta fe seldom COME ACTOSS accide pe vehicles are involved ey Shomical collisions are considered to he. etjeular accidents, similar conclusion, ag drawn here too. a "A large number of chemical reac, the balanced equation may contain four, ott moleculesfions participating in the reaction, ee examples are: me (i) N, + 3H, —> 2NH, (i) 4HBr + O ——> 2H,0 + 2B, (ii) 2Mn0;7 + 50,0," + 16H", +1 mt On the basis of balanced one molecularity of above reactions is 4.5" respectively. Since, molecularity cannot he gay three as simultaneous collision of all the reacting.» sane tingle step is not possible therefore, sucha” reactions proceed through a sequence of steps fa, is an elementary step and involves the simulin. collision of two or three species only. Such din: reactions which proceed through more than ows ‘are termed as complex reactions. Example The decomposition of hydrogen perasdev: is catalyzed by iodide ion in an alkaline mediunis as: 2H,0, > 2H,0 +0, ‘The rate equation for this reaction is found! Rate = - “14202! = jH,0) 11 5 dt 7 Evidence suggest that this reaction prooe* steps Step I. _H,0, +I-—— H,0 +10" Step II. H,0, +10-——> H,0 +1 +0 Conclusions if * Both the steps are bimolecular &*# reactions. * Species IO- is an intermediate. a * The first step, is a slow step and determining step. 29 mei! * The rate of formation of inte determine the rate of this reaction a * The detailed description of vatiO™ it reaction is ealled mechanism of 07 Scanned with CamScanner a Jecularity of overall reaction hi $e ter ance ‘in a.complex reaction.” + 0818 at steps of the given reaction are written Medill” .e experimental evidence like detection jased UPD “oof some short lived intermediates, ete 3 Ve ps ore known to proceed at different ates, oR curiot RATE EQUATION ‘us question arises here that how cn rate equation or rate expression of a now jch of the steps in the mechanism jon’ rate determining step. Tt must be Of) choosen termination of the rate law expression Asst Moactions is not an easy task. Its at oti requires: we mation about the number of moles of «Inform, consumed during the reaction. edie of the intermediates produced « A kno! the reaction and how much they during te during the early period of reaction, erate data to be supplemented by different * echniques so that the certain elementary steps ire verified to the maximum. inconple reactions the rate expression written ina the overall balanced equation has no athe bs (all. The rate expression for such complex satows eit on the basis of the experimental donly. [WUSTRATION Letus consider the gaseous reaction, 2N0,(g) + F,(g) —— 2 NO, F(a) «+ The rate expression on the basis of balanced equation is: rate = k (NO,]? [F,] +The rate expression on the experimental results is: ____tate=k [NO,] [F,] oaanathenatical expression written in terms ration of reactants, which actually the rate, is called Rate Law.” basis of 187 * Forany ‘hypothetical reaction, aA + bB——5 eC +dD ‘The rate law expression may be written as: rate = &(AI" (5) a ‘Tho numerical values of m and n are determined Experimentally and cannot be deduced from th balanced equation. of i eet ae ‘Tho values of m and of m and n may of may not be same as & as Wy oF may not be ‘The constant in the rate law expression is called ‘ate constant, velocity constant or Spe reaction rate, PS Rusledge Plas a Rate determining step A complex reaction proceeds through more than one steps. Out of various steps of the reaction the slowest step will decide the rate of overall reaction because the reaction cannot take place faster than the slowest step. Thus, the slowest step of the complex reaction is called the Rate Controlling Step or Rate Determining Step. Example 1 Thermal decomposition of nitrous oxide 2N,0 —> 2N, +O, On the basis of experimental data. The rate law expression for the reaction can be written as Rate = k[N,O]. ‘Thus, a possible mechanism of the reaction may be proposed as: Step]. N,0 #84?» N, +0 Step II. N,0 +0 —S###2» N, +0, 2N,0 ——> 2N, + O, Example 2 Thermal decomposition of dinitrogen pentaoxide 2N,0;(g) — 4NO,(@) + Oy ‘The rate expression for the reaction is Rate = k [N;O3] ‘Thus, the following mechanism can be proposed for this reaction. Step L N,0, 4 NO, + NO, Step I. _N,0,+NO, —“*-» 3NO, +0, Scanned with CamScanner 188 Do You Keoee? Reactions Where Rate Falls with Temperature Rise A small number of reactions which involve NO, the rate of reaction fatls with an increase in temperature. ’ Forexample, the reaction of nitrogen monoxide (nitric oxide, nitrogen (ID oxide) with oxygen 2NO (@) + 0, (@ — 2NO, (@) This is one of only five homogeneous gas reactions known to be third order: i Rate = k [NO}? [0,] The decrease in the rate of this reaction implies negative activation energy. The reason for this apparent peculiarity is that although the rate constant k does indeed increase with increasing temperature, the nism is such that another constant, the ibrium constant for one of the mechanistic steps, is also involved in the rate equation. This decreases with increasing temperature. ‘Solved Problems Based on Rate Law Dinitrogen pentaoxide decomposes as, 2N,0;——> 4NO, + O, Ifthe rate of reaction is expressed as ee = niigog, 22) — n,oy, A = hi0) then what is the relationship between the rate constants? Solution. According to the balanced equation, 02) dt o INO.) = En Fora hypothetical reaction, aX+by__, products, the rate is given a, 2% u isappearance of x. Solution, SFAADE to law of mass action, the a fe SrenesrateAXIDYD. As valueotas gags Ose “ oa : ~ de = BP typos or ORDER OF REACTION Ibis an important parameter jy every . It is always determined ck Tennot be writton from the Balanced persis 1 may be defined as,“the guy wa exponents to which the conceninan raised in the rate law expression » It must be noted that the molecules whose concentration does not change are‘e Mn for deciding the order of a reaction. coma For a hypothetical reaction, A. + BB Products if the rate law expression for this rea, Rate = A{A]* (B}* + The order of the above (m+n). ctons ig, eaction is eng, ‘The powers or exponents, ie, m and} no relation to the stoichiometric os and b of the balanced chemical equ * Order of the reaction with respetiosiy and that with respect to Bis n, * Ifthe sum of the power is equal toms reaction is called first order reaction Ifthe sum of the powers is two or threes reaction is second order or third et! reaction respectively. The order of anit can also be zero or fractional. Do You Kou? gg she Decomposition of ammonia over platinunct®# 18a zero order reaction Platinum is —— N, +3H, Rate=h (NH); order = zero 6 Sc Im the above decomposition reaction, mt" catalyst. At sufficiently high pressure ee Tha tiscompletely covered by the ammonit sc rofote, further increase in pressure me ic 's Rot change the reaction rate. eae “Pendent of concentration ands zero ote? Scanned with CamScanner -geucibonrcs” a ‘Some examples of reactions i tg are given in Table 4.1. ns with different orders (® For zero order reactions, Table 4-1. Examples of Reactions Rate = 2 = gaye Different Orders om" PAYNE ane a ea = hot of k= moll prone Order ET eden , Thus, the units of rate constant for zero order Toro ort renction are: mol I! 1, eS (Gi) For first order reactions, N,0 — Ny +120, | r= RENO} A a vl oat —> +1, [r= eine ° we ROLL a hemolict or k= 2. msec. —— 2P+3H, sk I oe aed PH a PH ° ‘. The units of rate constant of first order pesnre reaction are: sec", sgst order reactions 7 yen eases oeo, |ren moa 1 (i Por second order reactions, Rate= “= Hay? ,H,Cl— C,H, + HCl (C,H,cl) 1 mol L7 a 2N,0,—? 4NO, + 0, 1N,0,] 1 pecan 80,Cl, — SO, + Cl, {so,cl,) 1 coho +H) |= MC 1 7 = qnol gee ~Lmokt cert. ae +. The units of the rate constant for second 28Ra—> $He + 993Rn_ | r= k[Ra] 1 order reaction are: L mol sec’. pee (iv) For third order reactions, radizactive decay of un dz ‘Hable nuclei take place Rate = ~ =p [ays by first order kinetics dt , a Second order reactions moll = (mol ty9 X0+0,—+ NO, +0, be (NO}[O,] 2 see 2NO,—9 2NO+0, | r= 2 (NO, 2 or — a+, am 5 see mol .L> + The units of rate constant for third order ano) —> r= R{NOP ; Nol oe HINO} 2 reaction are: L? mol? sec". Third order reactions 20 +0,—+ 2NO, 7 =k [NO}{(O,] a Remember Fa +Br—92NOBr | r= k [NO}[Br,] 3 For a reaction of nth order, the units of rate constant tional order oo reactions de } & = hfeone}" 0 +l, coi, r=k(COPICL) | 2.5 ie Coc,—sco+a, |r=rtcocy | 15 or net _1_e = HCHO cH + a ad [cone time [eon +O |r=k(CH,CHOY? | 15 L 1 =x ¢ = [eone.]-" x time? ope OF RATE CONSTANT FOR REACTIONS Time Geone fF ~ Remedi Hm FFERENT ORDERS (mol LA)" x st, tate constant k has different units for | And for action of nth having different ene ee res mn of nth order, k has units of Scanned with CamScanner Comprehensive } 190 ty Differences between Molecularity Site the order art. eae, and Order of a Reaction nat Molecularity Order 1, It is sum of the powers of| the concentration terms in the rate law expression. 1. It is the number of reacting species under- going simultaneous collision in the reaction. 2. It is a theoretical concept. | 2. It is determined experi-| mentally. 8. Itcan have integral values |3. It can have fractional] only. values also. 4. It cannot be zero. 4, It can be zero. 5. It does not tell us anything | 5. It tells us about the slow-| about the mechanism of | est step in the mechanism the reaction. and hence gives some clue] about mechanism of the| reaction. 6. It does not change with |6. It changes with change in| change in temperature, | temperature and pressure. pressure, jolved Problems Based ¢ ate Constant and Orde: of Reaction (@ Draw a schematic graph showing how the rate of a first order reaction changes with change in ‘concentration of the reactant. (©) Rate of a reaction is given by the equation: Rate = KAP [B)? What are the units for rate and rate constant for this reaction, Solution. (a) Rate of a first order reaction is directly proportional to molar concentration of the reactant ie, rate « cone. So the graph is a straight line as shown in t the adjoining, figure: (0) Unit ofrate=molI-!sect 3 («rates ‘henge income time Unit of rate constant, Cone, —> rate ___ mol L' sec" = mol seont (AF(B) (mol imo 101? L? seem? k In terms of molarity, M, these can also be Unit of rate of reaction = M s-1 Unit of rate constant = M-2 gt written as hore M-= mol 1) @) HO, *8F + 2H" — 2419 tp % rate=k(H,Oyir) * ) €0+Cl,—> coc, rate =k (COP{chpi2 Gi) CH,CHO— cH co. rate =k [CH,CHoji tv) 2NO + Br, > 2Nop, Solution. () Rate = & (H,0,1(r] Order w.r-t. HO, ah. ST mop Rate =k (CO} [OL]: Order w.r-t. to CO =25 Order wnt Overall order = 2.5 Units of k= (mol L-)!-25 g Rate = k (CH,CHO}? Order wart, to CH,CHO = 1.5; Overall nes: Units of & = (mol L=)!!5 ot = mob yigt w@ Rate =k [NO}? [Br,] Order w.r.t. NO @ Gi) = (mol L741 51 = mol Identify the reaction order from a the following rate constant: @R=38.1x 10451 (i) k= 4.2 105 L mot = (ii) k= 6.5 x 10+ mol L*! 5. 4 mn. On the basis of units of rate constant thee" reaction are: (0 1st order (i) 2nd order (iii) zero order. “a Re NCERT INTEXT QUESTIONS ee ee CE yl Fora reaction, A+B— Pry =k (AJ! (Bp. What isthe Solution, The ord is sum of powers tration Tbe order of reaction is sum * Order=3 42205, Scanned with CamScanner go ‘he conversion of molecules Xu wea XtoYfotown sifconcentration of Kis neeased ots ie reaction X ~ ¥ follows secon order kinetics, idhe rate equation for this reaction will he: ~ sore, prere ate = AX? 1, then equation (1) can be written (@)? = ka? Rate; = sesconenuton of X is increased to three times, wo, W the Now we Hie equation will be: Rate = k Ba)? = 9(ka)® bere of formation will inerense by 9 times, Sar ition of dimethyl ether leads to the oe come and CO andthe renction rate 1 of tion of terms ofthe partial pressure of dim- io reer E> Rate = R(PcH,OCHs: js measured in bar and time in minutes, Na are na ynits of rate and rate constant? te the overall order of a reaction which has ression (ay? BI? (Rate = (AI? BT. (WCERT Textbook Solved Example 4.3) 1. Wen the reaction order from each ofthe following rate constants. (k=2.3% 105 L mol? (rex 104s. (NCERT Textbook a Solved Example 4.4) 1 bar, time? 4 (@ second order 2. (a) 2() 0.5 (®) first order. AWORK OF LAXMI PUBLICATIONS (P) LTD. g INTEGRATED RATE EQUATIONS We ‘onde? Already noted that the concentration hints is called differential rate equation. ways convenient to determine the 491, instantancous rate, anti insta an itis measured by determination af slope of the tangant at pine n concentration time plot (Fig, 43(0) 1.206), This maken edie to determine th rate lw and hence the oder of the tencton: In order to aunid. thin eifeuty we con inteqrate the diferent ate equation ta give a relation career measured experimental data, i.e, tions at different times and rate constant. © de tou Keow? * The integrated equation is used for: determining the fe aometany ho be asso + Tein heptincaealoting the ime in whieh the Tecan 20 or Go apie a + The integrated rate equations are different for the reactions of different orders. 1. ZERO ORDER REACTION ‘A reaction is zero order if its rate is independent of the concentration of reactants ox the rate of reaction is proportional to zero power of the concentration of reactants. R—P For a zero order reaction, dl Rate =- 22) - arr 4.2) ‘As any quantity raised to power zero is unity aR) ——=kx1 a or a{R) =— kat Integrating both sides, we get [R)=-Ft+1 4.2) where, Lis the constant of integration is initial ‘at t= 0, [R] = [Rly (where [Rlo concentration of reactant). Hence, I= Rl, Substituting this value in equation (4.2), we get TR=-ht+(R), or A= (Rly- BR) he +o - wi} SIGNIFICANCE OF INTEGRATED RATE EQUATION ({) All zero order reaction (i The value of can be evaluated if [Rly [R] at time ¢ are known. (iif) The expression, [R] = ~ ht + Rly is comparable to straight line equation, 9 5 ma * C,and cpampfore the value of k can be evaluated by graphical method. or (4.3) ns obey the equation 4.3. and Scanned with CamScanner eer reer r citiibiniendtimigs {@ At tow concentration of ammo ‘The above equation can be written as [R] = [Rp - Ht . On plotting a graph of [R] vs. t we get a straight line as shown in the figure below. The slope of the line is given as: slope =-# ‘The intercept on the concentration axis = [Rly Intercept Tino,t—> Further simplifying the equation, [R]=- kt + [Roy we get the rate constant, k as: Pe Re : Zero order reactions are relatively uncommon but they occur under special conditions. Zero order reactions generally occur in a heterogeneous system. Some enzyme catalysed reactions and reactions which occur ‘on metal surfaces are a few examples of zero order reactions. ‘The decomposition of gaseous ammonia on a hot platinum surface is a zero order reaction at high pressure, ont 2, w. ' 8) omer? Net) + SH); Rate = kINH,]? = k PF Kesuledge Plus Decomposition of Ammonia in the Prese nee in the nce of a ‘The decomposition of NH, on finely divide itinum surface is fire order when Soncentration (reenatnn ot NH,islow. Athigher concentration the reaction becomes zero order. At higher concentration of reactants (after certain concentration limit) the surface of the eatalyet ‘sflly covered and any further increase of concentration frente ot change thesia andthe retin becomes independent of concentrati becomes a zero order renction no” ad hence ‘The rate of reaction will be given as: rate =: ] ea (refer to unit V for details) Here, k, and k, are constants, ‘pe neglected as compared to uni Ue, fore rate = ky{NH,] igh concentration, we ne (® At high com neglect 1 ang 2. FIRST ORDER REACTION Tt us assume a simple hypothetical py reaction a8, R——~P If the initial concentration of R is (Ry rate constant and [R} is cone. at time :'y,°% differential form of this first order reaction wily = aR) A(R dt . Rearranging the equation, we get =dlR) = pat (R] vty On integrating equation (4.4), =a] - frat (RI -In[R)=kt+constant 45 ” isIn wi] Att=0, [R] =[R], therefore eqn. 4.5 reduces to form In [R], = x 0 + constant | or constant =— In [R]y | Substituting this value in Eqn. (4.5), wee# | In [R] =k t-In Ry We get, [Integral of or At=1n [R),—In [R] [R]y 3} kt=In— > "TRI Changing it to common log Rp | kt=2. Th 2.303 log TRI or 2.303 1, (Rly ta 7 Bl Equation (4.7) is the Integrated Equation for the first order reaction. oF ; se Physical significance of k. It repo 1 n of the reactant decomposed per unt! fractioy constant concentration. For example, if # t°® | Scanned with CamScanner ns aneCs js 0.00674 s-.Te ction is 0. means that mrgqam of substance would issocinte ot * 100 = 183 .6T sed initial concentration is kept conan, eon’, RIL = hk os pt constant, ay t+ or “alle @ Do You Krow?, Sa____——_ aT ——— > Equation (4. Dis Sah ritten in another form, ene a= (4.8) the initial concentration of R is ‘a moles TM tind x moles of it get changed to prodiet ager time t. Now, the concentration of reactant Hieftofter time t is (a~x) moles per litre. ie, [BJg=0 and (Rj =(a-x) «she equation is also sometimes written as: cace or N= Nye* (4.10) ere Cor Nis concentration at time ¢ and C, and Ny Geri itiel concentrations. Other terms have usual meaning.) « IF{R], and [R], are the concentrations at time f, and f, then above integral equation reduces to the form 1 _j, Wh _ 2303, ke = Bh G-) Rk @-4) [Rp (4.11) SIGNIFICANCE OF INTEGRATED RATE EQUATION (i If the initial concentration of reactant, [Roy (0) and concentration at time t, [R] or (a — 2) are laown then by putting their values in equation (4.7) (eqn. (4.8)], the value of & can be calculated. (i) The integrated rate equation also helps us ‘evaluate the order of reaction. This is done by ‘atstituting the value of [R], or (a) and [R] (or a ~ x) Tostssd (4.7) [or eqn. (4.8)]. If a constant value of ‘iebiained for different values oft, then the reaction first order, otherwise not. leat? use of above equation (4.7) or (4.8) in ‘lation of the order of reaction is illustrated in "4.8 in the solved examples. cats integrated rate equation is also used to ‘matin Yale of by graphical method. The (4.7) can be written as: ht gH tog hy gp FE tog Rlp—ls 2303 “2 TR] Z303 Jog [R]p — log [RI Tewe plot 1 i log IR} vs time or log 4 4, following seapha wou e 2 TRY time te Y Spe a eg] —> Fig. 44 (a) Jog Ro/(R]. —> time () —> Fig. 4.4 (b) In both the plots i.e., Fig. 4.4 (a) and 4.4 (b) we get straight lines whose slope would be given as: ig. 4.4 (@));slope= sk Gig. 440) slope=~ 3303 2303 From the value of slope, the value of # can be calculated. ‘The intercept made on the Y-axis would be log [Rly P. 2emenber ——_—__—_—_———— For the reactions of n'® order, nA—— Products ‘The rate equation of the form given below is obtained asia 1 2. “q—D|@- Fat ‘This equation is applicable for all orders except when fA.13) Scanned with CamScanner - 194 Comprehensive cre, TRY » a + Integrated Rate Laws for the Reactions of Zero and First Order S Order | Reaction | Differential | Integrated | Straight | Half-life | Unitsofe | type rate law rate law line plot | ° RoP | atyat= TR}-1R} | vst | IRI/2k 1 RP |apRyae=—AIR}] [R)=(R)geM or | In (R]vst | In Wk e= In (RIED) | order kinetics. ‘The rate law expressions for fractional order and second order reactions + All natural and artificial radioactive decay of nuclei takes place by first order reaction and follow fy, | Reaction Order | Rate Law Expression Integrated Form A—— Products v20r0.5| rate = {AIS Re (8)? (apt? | = HAE a 2A— Products 2 rate = R{Al "iat 2.303 [Bh {A luc = AIA] kt = — 2303 _ jog [Blo [A] | A+B— Products 2 rate = A(AJIB] Te -Be tas} ‘Solved Problems Based Order of Reaction ELEM 1 the decomposition of nitrogen (V) oxide 2N,0,——> 4NO, + 0,, following a first order kinetics, @ Caleulate the rate constant for a 0.04 M solution, if the instantaneous rate is 1.4 x 10 mol I-15". i Also calculate the rate of reaction when the concentration of NO, is 1.20 M. (iid) What concentration of N,O, would give a rate 2.45% 10° mol I! 1 Go hd Solution, (@) As the given reaction is of first order, therefore, rate = k [N,O,} or = tate [M= Cone. in mol L-!} 35x 1055, (i) Now if the concentration of N,O, is 1.20 M, then rate =k IN,O,} =8.5% 105% 1.20 .2* 10-5 mol L-t 5, (Gi To obtain concentration of N,O, when thersts 2.45 x 10 mol L71 s+, we note that Rate _ 2.45 x10 Fae eig TOT ONL or 01 WN,0, The rate of formation of a dina bs second order dimerisation reaction is 9.5 x 10° mol I 0.01 mol L-! monomer concentration. Caleulate tH constant. Solution. If the reactant monomer is A then the resis 2A— (A), (dimerisation) Since it is a second order reaction * Rate=AfAy? rate=—2 UAL, dle 2 dt dt a or 95x10 = 400.01)? or k= se = 0.95 L mor st. Scanned with CamScanner ‘pEMICALMONETICS 495 PSOE eperimentot date for the tion of NO; in gases phaseat 20 fever eats EERE The rate constant fora first order reaction Jecompost involving compound A waa found to be 0.082 min™!, When the - 2N,0a) > 4NO{e) + Of) initial concentration of A in 0.15 mol/litre, how long will it ‘Time (sec) Cone. of N, 1) take for the concentration of A to drop to 0.03 mol/litre? e PETE De Gol) Solution, 0 1.68 x 10 400 136% 102 800 114 x 10% 209 2300 195 102 2808 yg 915, _ 2.808 105 sa rao modo tirana eee Calculate the rates of reaction atthe tien given above. 2303 - to find 2908 0.6590 = 19.6 min. ( Rote constant (i) Order of reaction. solution CERT TNTERT QUESTIONS 7 AINgOs] Rate a= A first order reaction has a rate constant im] 0g | moe 5 SMB ae ET) NORTE fr moti? 18 10% How long will 5 ofthis reactant tahe to reduce (oe) | (mab) (mol Lot 1) oo |e ea 7168107 Solution. Given, Initial amount = a Final concentration = 3 ¢ 1 (163 ~ 136) x 107% Rate constant = 1.15 x 10 s** 400 | 1.36 «10% | > 400 240m107" ‘We know that for a Ist order reaction, = 3.38% 10% = 2808 jog (Blo 2808 gg 5 1a 14) x 107 io k (TR) 115 x10 3 mo, fie 2 eno 00) “ee 2303 x 0.2215 44.38 s = 444s. = 2.75% 10% 15x10 1 (1.14 ~0.95)x 10 fe a each. x1? [2 A24= 0.990210" | 9 99 x 10-4 ime required to decompose SO;Cl, eng 0% 2 1200 - 800 half of its its initial amount is 60 minutes. If the decomposition Esse xte isa first reaction, calculate the ate constant. Since the value of # is nearly the same. - The rate expression is, Rate = k[N,O,] Order of reaction = 1. For the decomposition of N,O, given as: N,0,{¢)—— N,O fe) + Zz Ose), ifthe rate law = hIN,Og) and k = 1.68 x 10 s", starting with 25 moles of N,O{g) in a 5.0 litre container at 298 K, how many moles of N,O, would remain after 1.0 minute. Solution. Initial cone. of N,Oq, [Alp in moleflitre = 33 = 0.5 molefitre According to integrated rate equation for the first order reaction (Ay __it ‘etal ~ Ba03 2 yg 0, $02 188107 «gorge 7 Tal 2.308 Jog 0.5 — log [A] = 43.76 x 102 Jog [A] = log 0.5 — 43.76 x 10-2 — 0.301 — 0.4376 = — 0.738 = 1262 [A] = antilog 1.262 = 0.182 moleflitre. = Solution. We know that for a 1st order reaction, 0.693 ha = Ttis given that typ = 60 min =, Saws 1. Consider the second order reaction, 2NO. —> 2NO +0,. The rate of decomposition of NO, at acertain temperature is determined to be 1.4 x 10-* mol L7* min at NO, concentration of 0.500 mol L-!. What will be the rate of decomposition when the concentration of NO, is 0.250 M and when the concentration of NO, is 0.125 mol L-!? 2, The decomposition of N,O, in carbon tetrachloride solution has been investigated 1,0, (solution) ——> 2NO, (solution) + 3 0,(g) Scanned with CamScanner Hi etetion has been found to be of nee aes : und to be of first order i Capa With a first order rate constant 62 eee aunts the tate of the reaction when () N04) ol L-} id (ii) = ” (i) What concenteation N39? eat eee ntrat P of 2.4 x 15S aeration of 10, would give a rate For the reaction 24+B+C—_, A,B+C; the rate Jaw has been determined to be Rate = A[AI[BF and the rate constant, k= 2,0 10- mol’ L? st. Determine the initial rate of reaction (9 with [A] =0.1 mol L*, (B]=0.2 mol L", [C]=0.8 mol 171. (i) Determine rate after 0.04 mol L- of A has reacted. 4. Show by using rate law, how much rate of renction 2NO + 0, — NO, will change if the volume of eaction vessel is reduced to 1 rd. 5. The initial concentration of N,O, in the following first order reaction N,0,@) — 2NO,(g) + ; 0,@) was 1.24x 10-? mol L-! at 318K. The concentration of N,O, after 60 minutes was 0.20 x 10-2 mol L-!, Calculate the rate constant of the reaction at 318 K. (NCERT Textbook Solved Example 4.5) 1. 8.5% 104 mol L-! min-t; 8.75 x 10° mol L-! min-!, 2 (i) 7.75 x 10° mol Lt s* (ii) 9.3 x 10° mol I=! 5 (ii) 3.87 mol L*1 8% 109 mol Ls, 3.89 x 10° mol Lt «!, 27 times, 5. k= 0.0804 mint, AWORK OF LAXMI PUBLICATIONS (P) LTD. a ——— PSEUDO FIRST ORDE! REACTION 2 The order of a reaction is sometimes altered by conditions. Consider a chemical reaction between two substances when one reactant is present in large excess. During the hydrolysis of 0.01 mol of ethyl acetate with 10 mol of water, amounts of the various constituents at the beginning (t = 0) and completion (2) of the reaction are given as under. Ee CH,COOC,H, + H,0 —> CH,COOH + C,H,OH t=0 0.01 mol 10 mol Omol O mol £ Omol 929ml 0.01 mol_-—0.01 mol ‘The concentration of water does nop much during the course of the reaction," &t Mey So, in the rate equation Rate = (CH,COOC,K, the term (H,0] can be taken as. consist mel ‘Wat, n thus, becomes Rate =k (CH,Co0c,H) : where k= i [H,0] and the reaction behaves as first order rag reactions are called psoudo first order reas Se Inversion of canesugar is another 5 order reaction. Peds, HH | CiHa20,, + H0 "> o,H,,0, + ox Cenesugar Glucose 00, Rato=h (CyzHy0,,] ey olved Problem Based on | seudo First Order Reaction ETRE Hytrotysis of meth acetate in an, solution has been studied by titrating the liberated aeny against sodium hydroxide. The concentration of the ag different times is given below: t/min 0 30 60 ” C/mol Lt | 0.8500 | 0.8004 | 0.7538 | omy ‘Show that it follows a pseudo first order reaction the concentration of water remains nearly constant (55 mal during the course of the reaction. What isthe value of itis equation? @NCERT Textbook Solved Example 4 Rate = k’ [CH,COOCH,| [H,0] Solution. For pseudo first order reaction, the reaction sia! be first order with respect to ester when (H,0] is const The rate constant k for pseudo first order reaction is: = 28 jog Shore =H 01 From the above data we note tmin C/mol 1 k/mol it 0 0.8500 es 30 0.8004 2.00410" 60 0.7538, 2002.0" | 90 0.7096 2.005109 oe It can be seen that i (H,O] is constant ae 2.004 * 10 min and hence, it is pseudo first onde We can now determine k from Scanned with CamScanner ¥ (85 mol L*4] = 2.004 x 10° mi 23.64% 10% mol? L mint OTHER METHODS For CALCULATING THE VALUR OF RATE CONSTANT RE CHANGE METHOD yet us consider a typical first order gas phase ot” ay Be) + 0@) etp be the initial pressure of A and p,the total seat hime't. Integrated rate equation for such wren ean be derived as: ‘otal pressure P= Py + Py + Po (pressure units) pu Po a2d pe are the partial pressures of A, B Po peatively. if atm be the decrease in pressure of A at time the mole each of B and C is being formed, the pa tad ne Treasure of Band C will also be-xatm each. ™ Aw — Be) + = Ce) wetzo pyatm— Oatm Oatm ji timet (P;-a) atm) x atm xatm sere pis the initial pressure at time t = 0. B= @j- 2) FEF A=P/ TE a 1 @p,—p) (4.14) ved Problems Based on fegrated Equation and lation of Rate Constan FRERIREYME Foro first order reaction taking place at 0K 2CO(g)—+ CO fg) + ls) The total pressure at the end of 401 seconds after 'ith pure CO(g) is found to be 0.313 atm. At the time pte raction is 100% complete, the total pressure is found 0.1645 atm, (0) Initial pressure of CO. ©) The rate constant of the reaction. Solutio, ‘After 100% reastionis complete only CO, isformed Hence initial pressure ‘ 1645 x 2= Ain because 2 molenof GO get comarca vs Co) 0 get converted to 1 mole of Now at 410 a ce) 2004) — a a 130202" 2e+ 2200100 at ‘Thin cone, CO = 0.3200 tn Cone ae 410 ,ee= 0.3200 2 0.207 stm j= 2208, 03090 2208 0207 * a0 sex 104 et Find the order of the reactionand the rate Gonatan for decomposition of ammonia na tungsten reat 856°C. The experimental dat te 302% x oss Total pressure, p (torr) | 228 | 250 | 273 | 318 | Time, t(oec) 200 [400 | 600 | 1000 | Solution. Change of pressure, Ap: 22 45 90 Time interval, Az 200400 800 4. oat 0.1125 0.1125 at Since rate of change of pressure remains constant, therefore it is a zero order reaction. ‘The rate constant, 011+ 0.1125 + 0.1125 _ 9 3417 torr s. aera 1. The following data were obtained during the first order thermal decomposition of N,0, (3) at constant volume: ‘2N,0s(s) — 2N,0,(6) + 0260) Total Pressure Hatre) 05 0512 ‘S.No. Time/s L 0 2 100 Calculate the rate constant. (NCERT Textbook Solved Examp | 1. k= 4.98 * 104 A WORK OF LAXMI PUBLICATIONS (P) LTD. _ Scanned with CamScanner Comprehensive RST Mi f 4.11 | HALF LIFE OF A REACTION It is defined as the time during which the concentration of the reactants is redivcer 19 itis tration 0} i Of the initial concentrate, U1 or the reaction. required for the comj Tt is denoted by typ OT fos (@ HALF LIFE OF ZERO ORDER REACTION ‘The integrated equation for zero order reaction is; IR] = IR] ~ Ht (R] Now at ty, [R] = oe (Rc [Rip _ Rp o typ = [Rlp~ B= [Ro =< (4.15) * wa = oe (4.15) From the eqn. 4.15 it is clear that the half life of a zero order reaction is directly proportional to initial concentration. oe typ [Bly The plot of ty us. [R]p is shown below: Initial cone, [F],—> ©0040 Kew? ‘The tors of 2er0 order reaction is 1.5 times fq At ty th i mies fre the concentration] = = [R], (i) HALF LIFE OF FIRST ORDER REACTION For first order reaction, we know that = 2303, (RI, ka S jog Ro. 2303) ¢ 8 Tgp oF t= 2808 tog a typ then (R]=71Rlp, when 2303, [Rh therefore, y=, log 7 = 2808 mook FR], logy ees o5, Ny It is quite clear from the above expr, the half life period or half change tin, order reaction does not depend een tng concentration of the reactants. Similar, required to reduce the concentration of the ai fa, any fraction ofthe initial concentration for he reaction is also independent ofthe initiat concent aig The half life of a reaction with nth order ig 1 ty: [Ro"~ +. For zero order reaction; t, = [R], For first order reaction; t,.«1 + For second order reaction; ty, = z lb 1 + For third order reaction; ty, « —~~ (RI? FP Keouledge Plus CALCULATION OF ORDER OF A REACTION WHEN HALF LIFE (t,,) IS GIVEN _ Now, by employing the above idea if we have different initial concentrations [R,] and (R,] for thess2* reaction and say (ty), and (t,,.), are their respectivebal lives then the order of reaction can be calculated 3s Cad en Od Ga mrt o Sh _ [B | Gade | = (2-1) flog R, —log R,] or nya 0 yah log (ty 2)2 | Tog Ry —log Ry | or = Ey gh-log thie py Tog Ra - log Ry Scanned with CamScanner PLOTS OF HALF Live: Zero order = ‘VS. INITIAL CON 199 First order CENTRATION : ‘Second order me ird order [> tin] oH tig tial initial conc. (a) intial cone, (a) ss Wa REQUIRED FOR DIFFERENT FRACTI RST ORDER REACTION ONS -@ ©” Kuowstedge Plus ora . fn addition to calculation of typ, the time site complete different fractions ofa first order The am ait ran ean also be calculated. Fae tine auste aeuesn 2 a For example, the time required to complete one- Say initial concentration = [Ral aint the reaction will be given as: RR, Cone. after 1st half life = {Rol 2308 jpg — 2 = 2808 Jog 3 = 2408 2 ty? 8 anal 3 = ate Now amount left after second half life similarly, the expressions for different fractions = Cone ef after fest alte tine to complete are given a8: _ ta Rl _ 1386 2x2 2 ( tyg OF tags = Similarly for third half live it will be = (Rol _ Rel (fgg OF toon = 2x22 Pt ‘Thus, for n half lives the amount left [A] = fl (i) fgg OF too = (Here n > number of half lives) 6°) gp OF fogay= So " oa fonow = Solved Problems Based on Half Life of a Reaction a i = ‘A reaction is 50% complete in 10 minutes. It is allowed 10 proceed another 5 minutes. How much a cu eomplctet heen 5 miter FOS zero order kinetics? tial cone. [Ry] = 100 mol L** In general, the time required to complete nth of a first order reaction is given a5 follows fim OF ty, OF ty75 = 2tos tars OF fs76 = Btys toazsy OF togsts = Mos Sams OF taggers = Stoxitoson OT "a0 | tyy= 2808 jog le _—_ = 2308 jog | Solution, Say the ini | KR) -(RL/ xt fp, half ofthis is consumed | 2 feet =5 mol L7 min* | bia | | = Stos Scanned with CamScanner ‘+ 75% of the reaction would be complete after 15 minutes. Caleulate the half-life period of a first order reaction from their rate constants given below: (@) 200s" (8) 2 min-! (© Syear', Solution. For the first order reaction, 1 = 0698 wok @ k= 2008 . 0.693 _ s * fin Pop 8485 x 10s o h=2mint ; 9888 5405 min © & 1782 year. EMME The halflife period of a reaction of first order is 100 sec. Calculate its rate constant. Solution, For a first order reaction, ty = = 0.693 tyo Now, ty» =100see k oss 3.93 x 10 sec. 10 A first order reaction has rate constant of 10 sec. Calculate the half life period for this reaction. Solution. For a first order reaction, typ = ae k= 10? sect 0.693 tia® Foe = 69.8 see. The ty, of a first order reaction is 60 minutes. What percentage will be left after 240 minutes? Solution. Number of half live: [Ab . a = Amount left [4] 0.0626 of [A], = 6.25%. PREMERA A first order reaction has k = 1.5% 10-6 ‘sec at 240°C. If the reaction is allowed to run for 10 hours, Comprehensive SETH | what percentage of the initial concentration changed into producte? What isthe halflife pering, hy tion? fii Solution. The first order expression is: i 2.303 t IRI | IR), = 1 and [A] =1-x = 1.5 X10 sect; t= 10 bp Substituting these values 2.303 10x 60x60 °° !ox k Let ee | 15x 10° x 36000 2308 = 0.0234 = antilog (0.0234) = 1.055 1.055 ~ 1.055x= 1 1055-1 1055 or = 0.052, ‘Thus, 5.2% of initial concentration has changed products. . ‘The rate constant of a zero order reacin 125 Mic! What will be the initial concentration & reactant if after 30 minutes its concentration is 007530 (RJ - RI 1 es = + (RJ) - 0.075} [30 minutes=8 GzllRel - 0.075] (80 min 1 Solution, For a zero reaction, k= Thus, 0.2: 0.25 0.5 = [Ry] - 0.075 [R,] = 0.125 + 0.07% A first order reaction has a se reaction rate of 10% «1. How much time will it take fo "4 0.2 M. the reactant to reduce to 2.5 g? Solution. Cone. aftern half ives [R] = "5 [Rh _ 10 22 sO = 20 eget or n= (R] ~ 35 693 typ of reaction = > = 693 sec 10% + Time required to reduce to 2.5 ¢= 693 x 2= 1886 sec. Scanned with CamScanner ecomposition of HO, isa first order 1 eit thealf change ime and st onder reaction, G josition from th: Tate constant, poste poss 1m the fact that the fracti fat and ato od OF 50 mimes 0.75, mn iitehange time for the decom, hal Position of N,O, g Bemvoure and is independent for the initial is £1 Megan of 8,0, Caleulate the specie reaction ene eater 20 percent comes, fate constant for first order reaction has been 4. Bate 5 48 x 10~ sec“, Calculate its two:third cpanee times fa" | isco substance A is mixed with an equal 4 Aunt of B. At the end of one hour 75% of a Ahas Sretod. HOW ‘much of A will be left unreacted at fhe end of 2 hours if the reaction is of first order Sn respect to A and independent of B? ren is of Bist oer wk reactant Avwith a : A peanstant of nin If we start with [A] = 0.5 mol spn would (A) reach the value 0.05 mol [7 (P.S.B. 1996; H.S.B. 1998) ‘action is 15% complete in 20 min. take to be 60% complete? (A.LS.B. 1999) «Art order reaction takes minutes or the initial + fone of 0.6 nol L7* to become 0. 4 mol L-!. How long willit take to become 0.3 mol L712 Theconcentration of a solution thatis initially 0.24 Mis reduced to 0.12 ‘Min 10 hours and 0.06 Min 20 eas hat is the rate constant of this reaction? a ‘Afirst order reaction is found to have rate ‘constant, «d the half life of the reaction. 5x 10" =}. Fine (NCERT Textbook Solved Example 4. 2) time required 10 Show that in a first order reaction, frempletion of 99.9% is 10 times of half-life ty.) cfthe reaction. (NCER Ades order Fe How long will it 7 Textbook Solved Example 4.8) L 25min; 2.77 x 10? mint. is aati R94, 8. 2004.62 sec. < ae é 5. 0.384 min. hes min. 7. 8.545 min. 10° rt or 1.9 x 10% 5. 41 251018 sec. 10. ta999 = 19 tas, a eG TEMPERA’ EPENDENCE OFTHE ere DEPENDENCE OF A REACTION ‘The rates of : ‘ates of almost a inde ». For example, In decomposition of N,O,, the time taken for ioe opens amount of material to compose is 12 mi 5. and 10 days at oon at 50°C, 5 hours at 25°C * In a mixture of KM is nO, and Oxalic acid H,C,00, potassium ‘permanganate gets lourised faster at higher temperature than at lower cmon emo ° In most of the cases it has beer is it nn found that the rate of the reaction becomes almost double for every 10° rise of temperature. This is also expressed in the terms of Temperature coefficient which is the ratio of rate jeonstanie) of the reaction at two temperatures differing by 10°. The two temperatures generally selected are 298 K and 308 K. Thus, Temperature coefficient _ Rate constant of 308K Rate constant at 298 K ‘Tho rate constants for the decomposition of NO, at different temperatures is given in Table 42. Flow the table, it is seen that rate constant at 373 K is 7.87 x 107 and at 298 Kis 3.56 «10%. This shows that for 25° rise of temperature of the rate constant increases by 43 times. Table 4.2. Rate Constants for the Decomposi- tion of N,O, at Different Temperatures ‘Temperature (K) 273 7.87107 298 3.56 x 10° 308 13.5% 10% 318 4.98 x 10" 328 15.0 x 10+ 338 4.87 x 10° nce of the rate of a ature the eee rely explained by proposed bY ish chemist, be accur’ 18). It was first tion (4. ry t Hoff but Swedis ‘Arrhenius ea! hemist, JH. van’ f but Swe¢ ee provided its physical justification and interpretation rar ra Scanned with CamScanner In the above equation, A is the Arrhenius factor or the frequency factor. It is also called pre-exponential factor. It is a constant specific to a particular reaction. Ris gas constant and E, is activation energy measured in joules per mol (J mol~). The Arrhenius equation can be best understood if we have a knowledge of the activation energy (E,) and the activated complex. ee ACTIVATION ENERGY ‘Areaction can occur when molecules of reactants collide with each other to form an unstable intermediate (Fig. 4.7). The intermediate exists for a very short time and then breaks up to form product molecules. Most probable KE — Threshold energy barrier Molecules with energy > threshold energy barrier will react Fraction of molecule Kinetic energy ——> Fig. 4.5. Distribution of energies of gaseous molecules. ‘The energy required to form this intermediate, called activated complex (C), is known as activation energy (E,). In simple words all the molecules in the reacting species do not have the same kinetic energy. Since it is difficult to predict the behaviour of anyone molecule with precision, Ludwig Boltzmann and James Clerk Maxwell used statistics to predict the behaviour of large number of molecules. According to them, the distribution of kinetic energy may be described by plotting the fraction of molecules (Ny/N,) with a given kinetie energy (E) us hinetic energy (Fig. 4.5). Here, Ne is the number of molecules with energy E and Nyis the total number of molecules. Only those collisions result in the formation of products which possess energy equal to or more than the certain minimum energy called threshold energy, Collisions of the molecules possessing energy less than. ry Contrctonie aay. | threshold energy do not form products, fy. reactants and products theres an energy hang must be crossed before the reactants are conyer, hig products. The energy required for crossing hs barrier is supplied by the kinetic enoyyy 1 molecules. a “The minimum extra energy over ang the average potential energy of the regg™ which must be supplied to the reactants ye them to cross over the energy barrier yore reactants and products is called Aci at energy”. Thus, ing ‘Activation energy = (Threshold energy) ~ (Average eneray ofthe rey or E,=Ey-Ey Energy evolved inthe reaction Progress of reaction ——> Fig. 4.6, Ilustration of activation energy and energy barrier involved in a reaction. The idea of activation energy and the eo" barrier involved in a reaction is represented graph! in Fig. 46, Each reaction has a definite value of E, a4 decides the fraction of total collisions whith effective. Obviously, if the activation energy reaction is low, large number of molecules can energy and the fraction of effective collision, /™ large. Such a reaction proceeds at high rate: oO other hand, if the activation energy is high, the?! be small and the reaction may be quite slow. Scanned with CamScanner tic fat reactions: activation energies are low, r low reactions; activation energie ae i 3 poe re ast whereas the reaction betw Ee 8 ist. tween CO and # 2NO+0,——> 2NO, , (Fast) 260,+0,— 20,,_(Stow) {rep COMPLEX OR TRANSITION STATE AC yas been pointed out earlier that during the ialveaction certain bonds are broken and certain ajoiot are formed. ‘The breaking of bonds requires i enereas ‘he formation of bonds results in the caer enere A eH Hou + [1.604 en f od 1 T elt Intermediate or Products Activated complex 4 4:1, Formation of activated complex during the reaction of Hy and Ty from HI. For example, in the reaction of hydrogen with sige to form hydrogen iodine, when a molecule of Jrimgen approaches that of iodine, H-H and I-I sear breaking and H—I bonds start forming, In se beginning, breaking of bonds predominates and | Side energy of the system starts increasing till it rmdir maxima (corresponding to threshold energy). Mer this, the energy starts decreasing because the trees of bond formation predominates and finally luis to the product hydrogen iodide. The a= of atoms corresponding to energy Se (iets energy) is called transition state ated complex. In transition state, the system Partial reactant character and partial product as shown in Fig, 4.7 and Fig. 4.8. ae difference between energy of the transition energy of the reactants is equal to activation 5, ‘anston state ~ Exeactants Exctivation 203 Progress of eacion —> Fig. 4.8, Transition state oF activated complex. “> Kuowledge Plus KINETIC STABILITY OF FUELS ‘The unusual stability of fuels even in the presence ‘can now be explained on the basis of concept of activation energy. In general, the activation energy of combustion reactions is quite high. When a flame is applied to the mixture of fuel and air, the ‘contents near the flame are raised to high temperature. ‘The supplied energy is enough to cause the combustion ofa portion of fuel in oxygen. The heat liberated by the combustion increases the temperature of remaining fuel fand the flame once produced is sustained. Thus, we ajise that many reactions which are feasible from the Consideration of free energy may be prevented from ¢ due to high activation energy barrier, Such energy barriers are blissful for the existence of life 0 Sarth because they provide stability to the reactants {ike fuels, contents of explosives in bombs, ete» which Mtherwise are unstable from the point of view of thermodynamics. This leads us to make a concluding thefament. that the reactants which 9% thermodynamically unstable | at ordinary temperature may not be kinetically unstable. EXPLANATION OF BFFECT OF TEMPERATURE Let us now consider the ‘effect of increase in temperature on the number of effective collisions. ‘On the basis of probability consideration Fig. 4.9 is drawn to give the eneréy distribution curves at is peratures T, and T, (where Ta T, + 10). Now as we know that the rise in temperature increases the Kinetic energy of molecules (~ KE “7 therfore, ‘istributi ve gets flattene’ and shifts the energy distribution cu! set fattone ion of igher energy region. a in the graph clearly reveals that the function Scanned with CamScanner ‘Twice as many molecules possess the threshold kinetic energy at 30°C Fraction of molecules a Kinetic Energy ——> Fig. 4.9. Energy distribution at different temperatures. Le, of molecules possessing higher kinetic energy, energy greater than threshold energy, as indicated by shaded portion becomes almost double and therefore the rate of reaction almost doubles for 10° rise of temperature. Thus, increase in the rate of reaction with increase in temperature is mainly due to increase in number of collisions which are energetically effective. ay ARRHENIUS EQUATION Arrhenius developed the mathematical relation between temperature and the rate constant on the basis of the observations from the large number of experiments. This temperature dependence of the rate constants is expressed algebraically as, k= Ag Fol T se(4.19) Equation (4.19) is called Arrhenius equation. Here A is pre exponential factor and is called frequency factor, E, is the energy of activation and T is the temperature in kelvin scale. The term ¢-./®T in the above equation is also called as Boltzmann Factor. Both A and E, arecharacteristic of the reaction, Another form of the equation which is more useful for calculations is obtained by taking logarithm of Eqn. (4.19), therefore, E, Ink=InA- pr (4.20) [- Ine=1) Converting to common logarithm, we get E 1 log = log A- —2— — log k=log A aoa (4.21) ‘The differential form of the equation (4.20) w.r-t. temperature can be written as: “he The equation is comparable ty y, equation. My The important point to note here i the value of E,, increases, the value of j 4,5 and therefore, the reaction rate decreases x", means that the magnitude of this effect of Thay temperature will depend on activation energy“ Evaluation of Activation Energy Arrhenius equation fy ‘The activation energy (E,) ean be evaluy, the graphical method. This is done by plotting? between log # against 1/T. By ‘The plot gives a straight line (Fig. 4.10) 45. | % | E, a slope equal to~ 57> . Thus, by knowingthey, of slope and gas constant R, the activation ener, | can be calculated u 7 E, =~ 2.303 RX slope | Alternatively, the value of E, can als determined by measuring the rate constants ¢; reaction at two different temperatures. If, andi: the rate constants for the reaction at temperatursi T, and T,, then logk vt Fig. 4.10. Plot of log & vs. wT. A ma log ty = log A~ 9303 RT, at) welt | Tog hy =l08 A- 9503 RT Subtracting Eq, (4.21) from Eq. (4.22) EB, f1_2 log ky — log k, = ——2— 2-4] 218 = 3303 lr: t Scanned with CamScanner 2303R | TT, | (4.25) ame results can also be achieved se equation (4.24) between limits T, and " 108 ey ] rus EQUATION—A JUSTIFICATION anil cTION RATE DOUBLING Fe now see that how the rate constant of a lotus rp the temperature is increased by econ aries “0 K. In general, for many of the (ivation energy (B,) is about 55 kJ mol ihe a Hence, om temperature. 55,000 _ (310-300) _ boo 5 * 73088514 (335%) = 0.30887 Ja ~ antilog 0.30887 = 2.08 = 2.0 coh ite clear that for majority of reactions ‘Thus, itis a najor erst eomstant doubles for 10° rise in temperature resroom temperature. The value of rate constant for the septonofritragen pentoxide, NO, ——> N,O,* 5 On 86x 10 at 25°C and 4.87 x 10° at 65°C. Caleulate the ‘7p of activation for the reaction (R = 8.314 JK-! mot-!). ‘tion. We know that dog = Ea _ [T= A 2303R | Tyla bg A8ix10% ___E, (338298 M46x 10°F ~ 2.303 x 8.314 ~ | 338 x 298 164075 = Ea _ |aeoml 19.15 [338 x 298. 205 19,16 .998 298 40 2148 19:16 398 «208 10 = 103684 J mot E, = log 140.15 103.58 kcJ mol. For a reaction, the energy of activation is zero. What ue of rate conatant at 300 K, if k= 1.6% 10° 1 at 280 K? (R= 8.81 JK"! mot!) (AJ. CBSE 1998) Solution, log 2 =—Fe— [AT] a0 (B= lon 2 = gaonn [Tm |=° (Beno! cs Beet on kya k= 1.6% 108s 1 ‘The experimental data for decomposition of N,0;[2N,0,——» 4NO, + 0.) in gas phase at 318 K are given below: Hence Timeeconda | 0 | 400 | 800 | 1200 | 1600 10 x(N,ogiM | 1.69 | 1.36 | 1.14 | 0.98 | 0.78 Timelseconds 2000 | 2400 | 2800 | 3200 107 x[N,0,IM | 0.64 | 0.53 | 0.43 | 0.35 (@ Plot (N,0,) against t (©) Find the half life period for the reaction. (© Draw a graph between log (N,Og) and t. (@) What is the rate law? (© Calculate the rate constant. (f) Calculate the half life period from k and compare it with answer (). Solution. (a) Fig. 4.11 show the plot of [N,O,] against time, tin seconds. 0.016 0.014 012 0.8.x 10 = 0.008 & G, 0.008 z 0.004 0.002 tyo= 14208 0 400 1200 2000 © 2800 3200 time in sec. Fig. 4.11. Plot of (N,O,] vs. time. Scanned with CamScanner (6) Half life period, t,, for the given reaction is the time during which initial cone. of [N,Oq] changes from 1.63 x 10 M to half this value ie, to 0.815 x 10? M and in the graph it has been shown to be 1420 seconds. (in graph locate the time against the cone: 0,0081) (© Graph between log [N,O,] and time, ¢ is plotted in Fig. 4.12 on the basis of data tabulated below: 41.800 10g N,0,——» ‘400 1200 2000 2800 9200 Time in seconds ——> Fig. 4.12. Plot of log [N,O,] vs. time. t 0 400 800 1200 1600 2000 2400 2800 3200 @) Since a straight line is observed when log [N,Os] is plotted against time, therefore the reaction is of first order. i _ - 0.295 Slope = -o* = = 0296 (0 Slope = 3303 ~ 14208 0.296 x 2.303 Rate constant, k= 2.205% 2.808 _ 4 35 o t= FES 95x 104 0693 0.699 (f) Halftime, typ 1428 hk” 4850x104 59 (The two values are almost same within limits of possible error). CERT INTEXT QUESTIONS What will be the effect of temperature on rate constant? Comprchensive SHSM Solution, The rate constant of a reaction is neay, with a 10° rig in temperature. However, th ext ay once of the rate ofa chemical reaction on termpergat pt by Arrhenius equation, in he (Where, Ais the Arrhenius factor or the frequg Tis the temperature R i the gas constant E, isthe, eneray) ligt ePanr The rate ofthe chemical dou, increase of 10 K in about temperature from 298 K cl Cag Ey Solution, Itis given that T, = 298 K nm, = (298 + 10) K= 308 K ‘We also know that the rate of the reaction, temperature is increased by 10°. dtd, Therefore, let us take the value &, = & and y, ty y= 2 Also, 8.314 J K mol Now, substituting these values in th ae low, ie. Arthenises, #3 toe 7.7 Z303R| hy hOB 10 Went oe Po asetaan mes or log 2= — 2 __|__10_] 2303 x 8314 | 298 x 308 | = jp, = 2808x8314 x 298 x 308, bgt 10 = 52897.78 J mol! =52.9 kJ mult DEGREE) The activation energy for the reac 2HI(g) —> H, + Ifa), is 209.5 kJ mot at 581 K. Calala the fraction of molecules of reactants having energy equil® or greater than activation energy? Solution. In the given case: E, = 209.5 kJ mol = 2 mol, T= 581 K, R= 8.314 JK" mol“ Now, the fraction of molecules of reactants bar energy equal to or greater than activation energy is 6 = = 209500 J mol” = loge= 2308 x 8314 JK mol? x58 = 18.8323 Now, Antilog (18.8323) Antilong 19.1677 = 1471x1079, — Scanned with CamScanner “aleulate the activation energy ofa reaction whos , Cale tatant is tripled by a 10° rise is 7 ihe ‘vicinity of 27°C. ‘temperature, vate constant of a second order reaction is 5:70 x 2. oor Le at 25°C and 1.64 10" mel eva wee Calculate the activation energy and the ius frequency factor, A. ate the activation energy for a renetion which s. Glatt rate when the temperature isralsed fom ‘te constant for the decomposition of N,O, at. 4, Thais temperatures are as follows: 25 35 «| 5 | « Ig 10° )1.05 x 10-]5.0x 1041.5 105]49x 105 Sawa graph and from t determine the activation Drovpy of the reaction. 4 Toerate constant ofa reaction at 800 K and 700 K + Mre002s~1 and 0.07 5 respectively. Calculate the falues of B, and A. “(NCERT Textbook Solved Example 4.10) ¢. Tefirstorder rate constant forthe decomposition sf ethyl iodide by the reaction 0,11 @) ——> CH, @) + HI @) 11600 Kis 1.60 10s". Its energy of activation is 209 kifmol, Calculate the rate constant of the reaction at 700 K. (NCERT Textbook Solved Example 4.11) 1, The number of flashes of a firefly changes with temperature as follows: [ #c ‘Number of Flashes per Minute Po 7 B 10 32, 20 35 31 38 47 42 82 44 108 Determine the energy of activation for the chemical is tsa ‘that leads to the flash. are eivation energy of a first order reaction at is 54 kJ mol“, In the presence of a catalyst ‘fevation eneray ofthe same reaction lowers to aie quot! at 27°C. How many times the reaction a pu itzees in the presence of catalyst at 27°C? rome ‘constant for the reaction of H, with i pm His 0.43 at 673 K I the activation energy \_ mx? ‘J mol-!, what is the rate constant at 1m RK? Ind ew Rgeae ene «envio ound fo gundnape ws ‘temperature changes fr 3 Calculate from 290 Kio 319K aca {he enery of activation fr sucha tection. 1. 84.96 bd mol B,,= 54.63 hd mot 3. 50.48 hI mol-!, 2.198 * 10°, 4. 103 RJ mot 5. By 18.28 hd mobs A = 161 Ig = 6.86 109 o 7, B= 88.74 bd mot 8. 55.08 times, 9.23.56. 52.86 bS mot! ‘A WORK OF LAXMI PUBLICATIONS (P) LTD. ay EFFECT OF CATALYST A catalyst is a substance that alters the rate of chemical reaction without itself being permanently chemically changed. Never state things like “it doesn’t react, just speeds it up”. It must take part in the reaction and it must change chemically, albeit on a temporary basis. A catalyst provides a different ‘pathway’ ormechanism that makes the bond breaking processes (orother electronic changes in the reactants) occur more readily. In general, Acatalyst speeds up a reaction, but it must be involved ‘chemically’, however temporarily, in some way, and is continually changed and reformed as the reaction proceeds. © Catalysts work by providing an alternative reaction pathway of lower activation energy. ‘Thus, the function of a catalyst is thus to lower down the activation energy. In sample words. greater the decrease in the activation energy caused by the catalyst, higher will be the reaction rate. In the presence of a catalyst, the reaction follows a path of lower ‘activation energy. Under this condition, a large number of reacting molecules are able to cross over the energy barrier and thus the rate of reaction increases. The energy profile diagram for the catalysed and ‘uncatalysed reactions are as shown in the Fig, 4.13. Where dotted curve represents the progress of uncatalysed reaction and solid curve represents the catalysed reaction. Scanned with CamScanner LUncatalysed reaction veg N Products ‘Activation Energies Fraction of molecules Progress of eaction —> Fig. 4.13. Potential energy curves for catalysed and uncatalysed reactions. For a general reaction of the type, A+ B—> AB the course of uncatalysed and catalysed reaction may be represented as: (@) Uncatalysed reaction: A+B— [A.....B] —> AB ‘Activated complex (©) Catalysed reaction: —> AB + C Product Catalyst [A+B] + =f Reactants Catalyst Activated complex Though the catalyst increases the rate of the reaction, yet it does not effect the state of equilibrium in case of reversible reactinons. It is because the activation energy for the forward reaction and backward reaction is reduced to the same extent. 2 Remember (@ Acatalyst is highly specific in its action. This means ‘that a catalyst may increase the rate of one reaction _, but may be completely ineffective in another reaction. (G Acatalyst may undergo intermediate physical change ‘and may even form temporary chemical bonds with the reactants butis recovered unchanged in mass and ay mmoson atthe end ofthe reaction ii) A catalyst never starts a reaction whi dene never starts a reaction which has no ten. (iv) Since catalyst does not alter the individual energi of reactants and products, ther iGetthe retcion remain unaligned? CoP AU ofthe The following example illustrates the rolo of catalyst: Reaction between oxygen gas and hydrogen gas is Satalysed by the surface offnely-divided platinunn 5 Comprehensive ae Wel yo-k tw cuennl ‘Acatalyst is a substance that enhances th reaction, There are two types of catainit a homogeneous and helerogencous. The betes’ catalysis is an important application in ayer catalytic converters. ‘Answer the following questions: (Asa student of chemistry, what chemicg, due to think occurs in the heterogeneous, in the converters? tly, (Gd Why the ei vehicles is important? te (G9 What environment values ae associated wit, use of such converters? COLLISION THEORY OF CHEMICAL REACTIONS Though, Arthenius equation is applicable nip | a wide range of circumstances but collision then | which was developed by Max Trautz and William Lay in 1916-18, provides a greater insight into the eneris and mechanistic aspects of reactions. It is based a kinetic theory of gases. According to this theory, ts reactant molecules are assumed to be hard spheresant reaction is postulated to occur when molecules cli: with each other. The number of collisions pet second per unit volume of the reaction mistut is known as collision frequency (Z). Anotherfats which affects the rate of chemical reactions is actvait, energy (as we already studied). POSTULATES OF COLLISION THEORY (@ A reaction occurs on collision of two molecilé only if they possess a certain minimum amount of en in excess of the normal energy of molecules. (i) The minimum energy which molecules ™* Possess before collision should be equal to or greater iat the activation energy. EXPRESSION FOR RATE CONSTANT ors BIMOLECULAR REACTION Say we have an elementary bimolecular rel : A+B—> Products es lace A°°*aing to collision theory the reaction He Place as a result of collisions between A and assume that the rate of reaction is equal to the ™*” Scanned with CamScanner 's unit volume per unit time multiplied ie ‘of collisions that have sufficient Telativg to overcome the energy barrier, e* (per pa ‘of molecules). Thus, the rate constant (or js ven ial sfeollisions or frequency of collisions (Z,,) e104 tor (given by Boltzmann factor, «ft poe the rate of a reaction would bo ally expressed 85: ue Rate = Zap- PO (4.26) pove equation predicts the value of rate he ab curately for the reactions that involve ass rly simple molecules but for complex a feegnificant deviations are observed. The wie Teason could be that all the collisions between A and B will not lead to reaction even if the energy requirement, is satisfied. It is because the colliding molecules should also have proper orientation. In simple words all colliding molecules having sufficient energy but no Proper orientation will bounce back without any Teaction. For instance, the reaction between two NOy molecules to form N,O, depends upon the collisions as shown in the Figs, 4.15 and 4.16. Similarly, it can be predicted that the, formation of methanol from bromoethane depends upon the orientation of reactant molecules as shown in Fig. 4.14. The proper orientation of reactants molecules leads to bond formation whereas improper orientation makes them simply bounce back and no products are formed. CH,Br+OH ——> CH,OH + Br Fs SD Improper | AA. : deme, SS SD Gu —> wo potucs Hy 3 is HOC_Br + OH H 7 H Proper. [Hye 3 i - Ga |HO-0--Br| —> HO—C-H + Br H’ “H 4 Intermediate Fig. 4.14.) = showing molecules having proper and improper orientation. ” pictorial representation (Figs. 4.15 and 4.16) is shown, for the reaction between two NO, molecules to ea,0, toillustrate the above idea Proper Orientation: $y =Fw=O% Collision Fig. 4.16 Scanned with CamScanner 240, Now, inorder to cope up with orientation Probability, a steric factor denoted by ‘P’ is introduced. ‘The value of P varies from unity to 10-®. Accordingly, Rate constant (k) = Rate of successful collisions = (Steric requirement) x (Rate of encounters) X (Minimum energy requirement) ie, (4.27) or REP. Zyq. eB +s(4.28) PF Reawledge Plus ‘The expression, k = P. Zsy . e“**/RT, derived on. the basis of collision theory can be compared with Arrhenius equation, A = A. e™/RT. In this expression, the Arrhenius parameter A is also called frequency factor and is mathematically given as: A=P.,5 AZM =P cuales A message is always displayed at the trate which reads Stop the vehicle engine atthe rede save the fuel”, ‘st ‘Answor the following questions: (® Why itis recommended to stop the vehicle en, the red light signals? *ertiney i) Ae a student of chemistry, suggest the chen rocese associated with the vehicle engine ne (did) What are the values associated with tnig mesg (Gs) Will you support the message displayed 1, ‘government or consider it to be violation of driving rights? as Scanned with CamScanner eS ERT Textbook Exercises Answers/Solutions ge exprensian forthe Following reartions poeta fection an the ‘dimensions ’ : gonstamt! ie fe) N01 + NO: Rate = KINO}? qiagr# 2H! —> 21,0 Is (a hate = KUO gion) —> HLL + COL (id Oe 2 k{CH,CHOl? grey) — Called + HCH wo rate = KICsHsCH- at qrder of reaction = 2. _ Rate _ ConcJtime teconstant = TN Cone? ee Gone. x time mol L's Lmols* seinensions ofthe rate constant hare: T+ mots? @ Rate=A0H,O,] iis Order w.r-t. HO; Order wat. P= 1- Order wart. H*=0. Overall order of reaction =1+1=2 constant, k= — = Cane.Aime (H,02)07] Cone” eS Gone. x time mol Ls =Lmols? Thedinensions ofthe rate constant, k are: L mols * (i) Rate = A[CH,CHO}” Onder wart. CH,CHO = 9/2 = 1.5 Overell order of reaction = 1.5, Rateconstant, = Rate ___ Conedtime _ iGH,CHOP? ~ Cone? Cone.”? xs ei) aes ‘dimensions of rate constant, k are: L!? mol!” s+ (Go) Reato= HC,HCH Ordor want. (CHf,01]= 1 Overall order of renction = 1 Rate constant, “Cone time __1_ figoH” Cone.” Hime = ‘The dimensions of rate constant, & are: s+ 4.2, For the reacti 2A+B+C——>A,B+C the rate= K{AIIB]? with k 2.0 + 108 Mie. Caleulate She intial sate of the reaction when [A] = 0.1 M, [B] = 0.2 3 aa IG) = 0.8 M. If the rate of reverse reaction is angligible then calculate the rate of reaction after {41 ig reduced to 0.06 M, eeretom In the reaction 24+B+C——> AgB+C, theres Solution. iC, therefore its concentration does not affect the rate of the reaction. Initial rate = AIAN)? 202M and k= 2% 10° M2 st x 10° Me? 5 x (0.1 M) x (02 MP =8x10°Mst From the equation 2A + B + C——> A,B + C it is cloar that whon 2 moles ofA are used then 1 mol of Bis used Gethe eame time. Therefore, when A has been reduced to $106 M (0.04 Mof A have reacted and hence 0.02 M of B). Thus Cone. of A left = [A] = 0.06 M Cone, of B left = [B] = 0.2M- 0.02] =0.18M Rate = MAIBE = 9x 10- Ms x (0.06 M) (0.18 9? = 8,99 x 10° Ms* decomposition of NH; on order. What are the rate of x 104 Ms. 4,3. The reaction of the platinum surface is zero production of N, and H, if fe Solution. The reaction for decomposition of NH is 2NH,—— Na + 3H Rate of reaction, dx __ 1 atNHy) 0g] _ 1 Ba) - 5 dt 22 «(dt dt =3 at and reaction is of zero order) (jis rate constant Scanned with CamScanner = 15 x 104 Ms, 4.4. The decomposition of dimethyl ether leads to the formation of CHy H, and CO and the reaction rate is given by Rate = k{CH,OCH,]* The rate of reaction is followed by {increase ie re in a closed vessel and the rate can also be expressed in terms ofthe partial Pressure of dimethyl ether ie, 2 Rate = K(Pen,oct, If the pressure is measured in bar and time in minutes, then what are the units of rate and rate constant? Solution. The reaction for decomposition of dimethyl enter can be written as: CH, -O-CH, —> CH, +1, +0 ‘The rate of reaction, dx) __ gICH;OCHs] _ dICH) _ dftig) _dfC0} (3) dt dt at at 312 = H(Pexocu,) ‘The unit of rate is: bar min“ or bar s ‘The unit of rate constant, kis: bar s“!/bar? = bar-ig-t k= —_tate (Pesz0cx, factors that affect the rate of a chemi- y 4.5. Mention th. cal reaction, Solution. The v: cal reaction are: ( Cone. of reactants (i) Temperature of reaction (itd) Presence of catalyst (iv) Nature of reactants (©) Surface area arious factors which affect the rate of chemi- How is the rate of ‘concentration of the reactant is, @ doubled Solution. Let the wart. A and the co} be written as: | Gi) reduced to 27 reaction, A, Bbe a 2nd ord i s-of AisamolL" then rateof rence rate of reaction can de de HAP = hat ‘Thus, rate of reaction will become fo ur is doubled. ‘ames bas (2) Similarly, when cone. of Ais reduce ied Rie, is $ then new rate of reaction, 4 a@) fafa as ae [3] pate, ‘The rate of reaction will become one f rate of reaction, arth 4:% What is the effect of temperature on constant of reaction? How can this tenner tH ry on rate constant be represented uantitannea lh Solution. The rate constant of reaction: increases wih, oftemperature Thisincroaseis generally tne for 10 Krrise in temperature. This is explained thenett : . ‘collision theory’. The main points of collision theory an follows: yay (@ For a reaction to occur, between the reacting species. Only a certain fraction of total cols, effective in forming the products. {G0 For effective collisions, the molecule mut ag the sufficient energy (equal or greater than thresholdenem as well as proper orientation, On the basis of above conclusions, rate of rating given by here Must be eg, Rate =/xZ (ohere fis theese collisions andisiad ‘umber of collisions per unit volume per st Quantitatively, the effect of temperature ontheneé @ reaction and hence on the rate constant k was propel! Arrhenius (1889). The ‘equation, called Arrhenius equatit is usually written in the form ke Actort a where A is a constant called frequency factor (is! gives the frequency of binary collisions of the rest) molecules per second per litre), B, is the energy of activil# Ris gasconstant and Tis the absolute temperature. Tx! ©" gives the fraction of molecules having enerey 19 oF greater than the activation energy, E, [The energy of activation (E,) is an important asitis character o 3 istic of the reaction. Using the above its value can be calculated, ‘Taking logarithm of both sides of equation (h"*" Scanned with CamScanner Ea RT, ii) Eq MARE, mc) a unteotns ‘equation (i) from equation (ii), we got phe-oet (& = 22 _ Fy Ink- =" RT (RT) RT RT, Be s(d a) =F M=T) myo R (i BR ta) the values of the rate constants ky and 'T, andT,, the value of E, can owing smperatures st order hydrolysis of ester in water wre obtained : 30 60 90 oss 0.81 0.17 (0.085 the average rate of reaction srval 30 to 60 seconds. do first order rate constant do fir ins results We! ws pater (pCaleulate the time inte {ip Caleulate the pseu tee hydrolysis of ester. nn.) Average rate of Fea pw 6d second is given by, _ be _ Cp _ 0.17 - 031 Average rate = "ap" gp 0-30 .ction between interval of time O14 __ 0.90467 =— 4.67 x 10° x Ms 30 Minus sign shows that the rate sgrihtine as conc. of ester is decreasing wit of reaction is decreas- ith time. 213 The ne of rst order eva ‘confirms that reactions: ‘The actual valu values ttl value orate constant isthe average of three 4 rate constant of reaction, net ta +h _ (191+ 195 + 2.06) x 10°? a £1.97 107 st, 4.9. react 9.4 mis first order in A and second order in B. ( Write differential rate equation. (ii) How is the rate affecte: : Aeeein fected when the concentra- (iii) How is the rate affected when the cor tion ofboth A and B isdoubled? she concent Solution. (@ Differential rate equation of reaction is: dz @ 7 Mar (i When conc. of B is tripled, it means cone. of B becomes [3 * B] (BP = ALA? ., new rate of reaction, F— dt (AIBBP -ouanor=+(4) ise, rate of reaction will become 9 times. led and that of Bis also (2B). rate of reaction, 2 = MA)(2BF = SAAIBP ‘reaction will becomes 8 times the F ween A.and B, the initial rate of reed for differential initial i.e,, the rate of ate asin (). 4.10. In a reaction bets (i Pseudo first order rate constant fis given a5 420 vas meas = 288 ho veneentrations of A and Bas given below: arr ‘eos initil cone. and a — x cone. after time ¢- [Ammer Lc? [0.20 0.20 00 Ter, a=0,55 M, Let us caleulate the rate constant = ‘ : i 2 a 0.30 0.10 0.05 te diferent instant of time, 30, 60 and 90 seconds a5 Binol L - rafmot tite! | so7x ioe] 507" qos] 7.6% 10 What is the order of reaction with respect to A y= HOLX 10s 21.95 x 107s" dB? . a Aisxand we. . Bis Solution y. Then rate of n, Let the order of reaction w.r-t- - eof reaction can be written as: rate = MAPBY From run 1 and 2, we can write, 5.07 x 10% = k{0.20) [0-307 5,07 x 105= {0.20} (0-10 Scanned with CamScanner Dividing (@) by () we 6° 07x10" _ HO: 20}10.10)” Box 10% Ho20}"(080P =0 Ps Fiomran Zand woces wee x 10-5 = {0.20}* [0.1 . “er (02072 Win (= oy (o.05'= MOAOP x1 Ait) Divide Cn by Gi we get 7.60 107° _ A001" [ss 507 x 10% Ho.20" = 0.2. r= = 1.5 on solving == 05 mt « ‘Thus the order of reaction wrt. Ais 5 and w.r.t. Bis zero. ; 4.11. The following results have been obtained during the kinetic studies of the reaction: 2A+B—>C+D or Knowing x and J, rate law can be written as | Rate = RAJ[BI? Onder of reaction wrt, A=1. | Order of reaction wrt. B Overall order of reaction Substituting the values of initial rate of formats experiment () in (equation v) we can write, 7 6.0 x 108 mol L7 min“ = (0.1 mol L-[0.1mo1 e — 6.0 x10 M min eS = 80x10 Mini 1x10 M =6M*s* 4,12. The reaction between A and B is first order wig respect to A and zero order with respect to B. Fil the blanks in the following table: Determine the rate law and the rate constant for the reaction, Solution. Let rate law of reaction be, rate=[AP[BP where x and y order of reaction wart A and B respectively From experiments I and IV, we can write, 6.0 x 10-9 = R{0.1 [0.1 24107 = #(0.4)* [0.1 Dividing (i) by (), we can write, 24x10? (ee = O41 60x10 o aaa « From experiments (I) and (II), we can Write, 7.2107 = (0.3) @.2y ' 2.88% 10 = k0.3)40.4)7 iii) (iv) [Exper- | aj | [By Initial rate of Experiment (Aya [BJM | Initial rote iment formation of MM mint DM min — I O21 On 2.0% 10? I o1 | ot | 60x 10% mol L? min? 0 - 02 40x 107 i 03 | 02 | 7.2x10-%molL7 mint mm 0.4 Od - | mr | 08 | 04 | 2.88% 10% mol L-! min+ IV - 0.2 2.0 x10" w | o4 | oa | 24%10%molL4 mint Solution, Rate law for the reaction is given as, Rate = k{A]}[B]? = AIA] From experiment 1, we get: 2.0 X 10-* mol L=! min“ = {0.1 mol L*) 2.0 x 10°? mol L7! min” 0.1mol L™ or Rate constant, # = 0.2 min“. (0 In experiment IE: rate = k{A] Rate constant, k= rate k 4.0 x10 M mi . 0.2 min’ (@ In experiment TI: Rate = k{A] 0.2 min“ x 0.4M. 0,08 M min-! = 8.0 * 1 or [4] 020% poe Scanned with CamScanner experiment IV: 9 Pee = AA] = 20x10? M min“ 02 min pos the completed table i: (Al B) Init mol Lt mol It ee at o1 | 20x10 a2, 02 | 40x102 04 O4 8.0.x 10% oa 02 | 20x102 tate the halflife period of a first order reae- ue vate constants given below: en 200s? (6) 2 mint —aajon For the first order reaction, ae _ 0693 tp? (©) 5 year? 693 f=" 200 0.693 typ = —g = 0.8465 min 0.693 (keayear? 6 hn ULThe halflife for radioactive decay of MC is 5780 y. Anarehaeological artefact contained wood had only sof the “C in a living tree. Estimate the age of the spe Sdution. Given, ty» = 5730 years; - = 80% = 0.80 0 @ e207 = 8.465 x 10s @a= = 0.1732 year Decay constant, j= 2693 _ 0.693 tyg 8730 tng Al ttioactive nuclear decay are first order reactions, 21x 10+ year? + Decay constant, log Xe 2303 | 10 7 ee 2.308 11x10 Aref sample = 1845 years. — x 0,969 = 1845 years 218 for der ‘composition of al in gas phase at 318 K 9.64 | 0.53 | 0.43 (2) Plot (8,0, against « (6) Find the half ite period for the reacti {0 raw a ereph betwen in (0) 480 («l at is the rate law? “ . (€) Calculate the rate constant. (f) Calculate the hal fe compare it with answer (bs "roa from Ke and Solution. (a) Fi time, tim seconds #17 Show the Plot of T8084 aeainat Fig, 4.17. Plot of [N,O,] vs. time. (b) Half life period, typ for the given reaction is the time during which initial cone. of [N03] changes from 1.63% 10? M to half this value ie, to 0.819 * 10 M and in the graph it has been shown to be 1420 seconds. (dm graph locate the time against the cone. 0.0081) (©) Graph between log [N,O,] and time, t- t og yp N05) 0 = 1.7918 400 = 1.8665 800 1.9431 1200 = 2.0315 1600 = 2.1079 2000 — 2.1938 2400 = 2.2757 2800 = 2.8752 3200 — 2.4559 Scanned with CamScanner 1.800 1.900 -2100 ee g-2200 % -2.400 -2500 2600 “4007200 2000 2800 2200 Time in seconds —> Fig. 4.18. Plot of log [N,0,] us time. (@ Since a straight line is observed when log [N,O,] is plotted against time, therefore the reaction is of first order. = ah 0.295 © Slope © 2303 ~ 1420s or = 4.85 x 104 st 0.693 0.693, = 1428s ke 4.850 x 10 (The two values are almost same within limits of possible error) 4.16. The rate constant for a first order reaction is 60s", How much time will it take to reduce the initial concen. tration of the reactant to its 1/16th value? Solution. Rate constant of reaction, k = 60 5"! 4, hans Rate constant of first order reaction is given as, when 15/16th of reaction. Hence, than 2.303 og 16 = 2303 log. So * 120416 .046 5 = 4.6 x 10-2 sec, 4.17. During nuclear explosion, one of thi et sar wanallicorares am ofthe products in the bones of a newly born baby instead of ‘calcium, how much of it will rem; after 10 years and 60 years if it is not lost metabolically, Solution. All radioactive decay are first order ‘Processes, decay constant, : (@ whent given, No=1 microgram 1 * 104 gy, After 10 years, the amount of Sef is May 2303 | 1x10 ‘ 1x10 _ 10x 0.0247 Jog = 39g = lore * or 1210 = Antilog 0.172 = 1.1269 _1x10% or N= "Trad = 0-7842 ug (i At C= 60 years, the amount left wil ie 2.303 1x10% o0247 Ty 110-6 _ 60x0.0247 log N 2303 = 0-6435 1x10* o _ N Antilog 0.6453 = 4.400 1x10 N ~qa00- ).227 ug Thus after 10 years and 60 years, 0.7842 npxi 0.227 mg of Sr will be left as undecayed. 4.18. Show that time required for 99% completa twice the time required for the completion of reaction. Solution, For any first order reaction the time req! completion of a particular fraction is given as: t= 2803, (Alo ee TA : (AU terms in expression have usual nee Accordingly, = 2808), Al _ 2808 91 foo" “E'S Sorta a 2303 [Alp __ 2309 p10 Simila Jog <8 = 2808 ag "iY faso= “Ee Tay 2 fogo 2308) 9 ok | 1? “ foo 1°81 5 a95 * Tog r fo00=2 tog td 4.19. A first order reaction takes 40 min for 30" Position. Calculate ty. w 2.303 tog — fg = 2308 jog [AD - 28 op 7 Solution, k O7[Al = 2303 FX 0.185 Scanned with CamScanner 2.303 Re 90 x 0,185 = 8.92 x 10 min, ‘ 0693 __0.699 Pe = g92x109 | 7768 min. composition ofazolsopropane to hex pe dees K, the following data is obtained. aa “GuN'= NCH (CH), —— Na+ Cally (mm of Ha) 35.0 63.0 ate the rate constant. cate se pose pis the initial pressure at t= 0. solute oles sf avoisopropane decompose and the NCH(CH,), —> No + CoH, Pm iy 0 Prd 2.303 ‘eo= “360 22.17% 10% s* Silly, _ 2803 720 2303 35 kn= 09 OT = 2,235 x 10% s x 0,699 during the first {1M following data were obtained (g) at constant xe thermal decomposition of SO,C1, ine. $0,Cl, (g) —— SO, Cl,(«) + Cl®) Times! | Total pressure/ i atm ja 0 0.5, a 100 0.6 tag eta the rate of reaction when total pressure Solution. Suppose initial pre of $0,Cl, is p, and its pressure decrease by x atm after time ¢ then, hen 4,22, The rate at various temperatt __ 804Cl,(@) ——> 80,0 ) + Ch Initial pressure (at¢=0) 0.5atm 0 Presmure(attime=) OS—x Prag At time (= 0,6—2+2+x=0.5 + xatm B42 or ¥206-0.5=0.1 atm 5 atm and p (at time 100 ree) 0.5-2=05-0.1 = 04 atm (a Evaluation of 2.808 hua Initial Pressure Pressure at time t 2303 x 0.0960 = 2.28 x 109 5%. (& Rate at Py = 0.65 atm p80,Cl, at total pressure of 0.65 atm =0.5-0.15 0.35 atm Rate = k (pS0,Cl,) 2.23 x 10° x 0.35 7.8 x 10+ atm constant for the decomposition of NiO, ure is given below: T/C o ~ | o || 10° net | oove7| 170 | 25-7 [8 [210 the values of A and E,. Predi Draw a graph between inkand VT and calculate ‘ct the rate constant at 30° and 50°C. Solution. To draw the plot oflog K versus UT, we can rewrite the given data as follows: — 11K) | 273 293 313 333 353 ir |0.003663] 0.003413] 0.003195 (0.003008 | 0.002833 Tog k | 6.1040] - 4.7696 — 3.5900 | -2.7496 | - 1.6996 the figure 4.19. From Draw the graph as shown in graph, we find, ed Bo Slope = 99074 2.303 R 2.4x2:308 XR, 0.00047 49.303 x 8.314 J mol ~ 0.00047 = 17.689 kd mol ‘Activation energy, B= Scanned with CamScanner 218 E, We know that log k= log A~ 3358 Rp =(-—»_ « weh=(~ gaat) # log A= value of intercept on y-axi -1+7.2)=6.2 og k ——> ; which is equation of line j A ) tor compaeitnthy=mete hich is equation of ne ang, on log by e-1-C7 igi Hg iy '00028° 0029” o.0031 00088 00085 00087 “t ——> J $ Fig. 4.19, Graph of log & vs. W/T. Frequency factor, A = antilog 6.2 = 1585000 =-46184+17.21 1.585 108 collisions s* 6184 + 17.21 ‘The values of rate constant i can be found from graph aero as follows: log A = 12.5916 pr UT | value oflogk| value of Taking antilog of both sides Bas. (from graph) A= Antilog 12.5916 303 | 0.003300} -42 | 6s1x10561 a 323, | 0.003096} -28 | 1.585x 10-964 mets 4.23. The rate constant for the decomposition of hydrocarbons is 2.418 x 104s at 546 K. Ifthe energy of activation is 179.9 kJ mol", what will be the value of pre-exponential factor. ion, log A=logk+—E2— = Solution. log A= log k+ 5353p =log 2.418 x 10 5-1 179900 J mol“? 2303 x 8.314 JK mol x 546K 7389 decomposes in acid solution into glucose rose socrore deer ding to the first order rate Inw, with Uripactose #CCOP™ ction of sample of sucrose remains ofp b. What fr a shours? : oe ora fist order reaction, it 0.693 988 0281 1.0 mol (.0=3) Hence, Tr = antilog 0,802 = 6.345 = ' 126345 (1-2), 2= 5548 = o.s42 +: Cone, after 8 hours = 1.0 — 0.842 = 0.158 M. | {i The decomposition of hydrocarbon follows the qution x 10M) #8000 0/7, Calculate Ea. Sion, Aecording to Arrhenius equation, k= A. e~Fa®? mute the given value of k= (4.5 x 1085-1) «2600 ¥7, or £4 =28000K Yes, Ba = 98000 1 B Ba= 28000 K x R = 28000 K x 8.3145 K-! mol". 282792 J mot = 232.79 kJ mol". Therate 4 ‘constant for the first order decomposition isin by the equations 514.84 1.25 x 10¢K/T Caley "ontuette Ea for this reaction and at what ill its ty, be 256 minutes? Solution. () Ono mpari ith the given value of & Arrhenius equation, k= A. 2="T aoe Daan = 1-25 10* ‘Ea = 1.25 x 108 x 2.303 x 8,314 J mol“! K+ = 239,339 J mol-! = 239.34 kJ mol. (Gi) For a first order reaction, k= 993. or ta ee 0.693, k= 0803 4, Beaxey = 851% 10%, Given, log k= 14.34 1.25 « 10° KT or og (4.51% 10-5) = 14,94 — 225% 108K T a = 4.85 = 14.94 125% 10°K 0 or 14.3444,34 = 225% 10K 25 x 10K or p= AWK _ Ta59 7 8088 K. 4.28. The decomposition of A into product has value of Kas 4.5 x 10°s? at 10°C and energy of activation 60 ki mol. At what temperature would K be 1.5 * 104 12 Ea_[%-T, Solution. We know that, log #2. - 2a _| Ta Ti) ° Si wal ‘Tih | tog 25% 10* 60,000__[T, - 283 45x10" 2303x8314| 283T, . Ty = 283 0.5228 = 3133.62 [ a | or 05228 _ T,- 283 313362 2837, 1.67 10+ x 289 = 2=288 by 0.0472 T, = T, ~283 T, — 0.0472 T,, = 283 0.9528 T, = 283 Hence, T, 297K. ~ 0.9528 4.29, The time required for 10% completion of a first order reaction at 298 K is equal to that required for its 25% completion at 308 K. Ifthe value of A is 4 x 10's, calculate k at 308 K and Ea. 2,303 |, [Aly _ 2.308 1, 100 age Tal 2803 ,,., 100 Similarly Bygs=—p 1°85 Solution. O wii) Scanned with CamScanner As time required for 10% completion = time required for 25% completion log 100/75 _ 0.1249 = 01289 22.733 %ao3 Tog 100/90 0.0457 According to Arrhenius equation Now, or log 2 -—Ea_{2_ 1 7 %, 303K |T, Ty log 2.723 Ea [ze = 3] 2.303 x 8.314 | 308 x 298 Ea: 0.436 x 2.308 x 8.314 8.36 ~~ 4089x104 - losoxjo> 1089 x 10° 1089 x 10 = 76658 J mol" Ieggg =A. FART 1 Jog Bygg = log 4 x 1010 ee anes 2308 * 308 76658 = 10,602 — —_76658__ 2.308 x 8314 x 308 or aos a0tlog 2.396 = 40255 a) 4.80. The rate of reaction quadruples when, ture changes from 288 K to 318 K. Caleta n'y, of activation of the reaction assuming thay change with temperature. toa Solution. 1, = 203K; T,=313K According to Arthenius equation, tog FE = Ee (a) 23038 (Ty, Ea=2,303R x (Joes | = Ty | TT _ 293x313 | els ie oe t Now, yoy ~ 313-293 and log 2 = log 4 Hy 8 7 = toe 4= 0.6021 + Ba= 2.803 x 8.314 JK "Imo x 4585. 454x050 = 52863.33 J mol" = 52.86 kJ molt, Scanned with CamScanner Solved Questions es) : Involving Higher Order Thinking Skills (HOT: ‘S) phat typeof reactions the rate constant shall have Pins the rat of reaction? at has eo Zero order reaction. the rate of @ chemical reaction do not remain i out the reaction? Isitposileto have rections 1 orm rate? unio js Because rate of reaction depends on . anbich changes with passage of time. neon of r0 order proceeds ata uniform rate, sis possbl to have reactions of 4th order. Give one B, freon having an order of, ifany. Ma Usualy the order of reaction does not exceed thre, sation ge simultaneous collisions of 3 or more molecules pissobecau ple. However, reactions with an order of 4 or favour cles very rare. An oxample of reaction with order ott gal to fr i 4KC10, — 8KCIO, + KCl, “Achemical reaction has zero activation energy. What necfet of temperature on rate constant of reaction? sation. According to Arrhenius equation A- Ea ReA~ ap Since, B= zero REA Hence, the rate constant of this reaction is indepen- fete temperature. El Achemical reaction RP, has halflife (ty) depending Shape of concentration of reactant. If the concentration necant is increased by 5 times then how many times the acon rate increases. ‘Soton, For the reaction, R—> P tgs RP. Ingeneral, ° foe (Here, n = order of reaction) in , n= order of reactio & For the gi ‘i daenagestt tine istinn, 2-0 action is zero. For zero order reaction, rate = k(R]° Hence, the rat i i senna th Fain independant of concentration of WERE Con a reaction have zero activation energy ? Solution. If E, = 0, then the expression, kaA.e PRT oA aA ___ In other words the rate constant becomes equal to collision frequency. This implies that every collision results into a chemical product. This cannot be true. HEB State the rote of activated complex in a reaction and state its relation with activation energy. Transition state Heat of reaction Progress of reaction Potential energy diagram of an endothermic reaction. Solution. For energy reaction (irrespective of endothermic or exothermic nature), reactants have to cross an energy barrier to change into products. When reactant molecules absorb energy, their bonds start loosening and new bonds start forming between them. This highly unstable transition state between reactants and products is called activated complex (represented in figure by AB*). This activated state is a transition or short lived state and the activated complex may ‘change into produets or back into reactants as shown below: Scanned with CamScanner a 208 (products) , 6 Aeon B (Coactants) (activated complex) tivated complex Activation energy = Energy of act om Avorage energy of reactants, Tho magnitude of activation energy depends on nature of reactants. HEE A catalyst only changes the speed of a reaction whereas a photosensitizer initiates the reaction, scinmto in a Solution. A catalyst does not chemically participate reaction. It simply provides a path of low activation energy. On the contrary a photosensitizer acts as the enorgy carrier for a reaction and hence helps in initiating a reaction. HEIN Tre progress ofthe reaction A <—=™ nB with time is represented by the Figure, Determine "Coo Cone. (mot typ. Onan eardee @nea Time (hours) —p @ the value of n, (i) the equilibrium constant K. (iti) the initial rate of disappearance of A. Palution. (At equilibrium, the cone. of A falls from 0.6 mol 1" to0.3 mol 17 and cone. of B rises from zero to 0.6 mol L71, Thus, 0.3 mol L* ofA on decomposition gives 0.6 mol LofB n=2 ti BP _ (06)? k= EE - 06" _ @ me SE ae (ii) Initial rate of conversion of A aa) oa = =0.1 mol hrt, em concentration of a reactant followi oy lowing first order inetics, decreases from 19 8 10 2.5.8 in 60 minutes, What is the rate constant ofthis reaction? M2, be n0. of half lve or Hence, tig 27 30 min For a first order reaction, j= 2693 _ 0.693 ty 309-0231 Numerical Problems Tnvolvi Higher Order Thinking Skills (Hor, he rate constant of «stro order rain mol L-! ¢". Starting with 50 moles, ealeutng tet minutes in which the concentration decrease a Solution. For a zero order of reaction x = y {e=hangein concent x _ 50-10 _ Bat 740% Bs iz 40 x 10° 667 min, 60 HER Find the two-third life, tyy of a first order et which k= 5.4 x 10-41, Solution. The rate constant & for first order wars expressed by relation, 2.303 ace 303, a Substitute, 20 tay R= 5.4 x 10M et 22 2.303 54x 107 as 2303 7 Ba * 10M (log 3)s = 0.4265 x 0.4771 x 10" we Tworthird life of given first order 1 2.085 x 1013 5. oa Starting with 40 g of a radioactive so the original amount is added to it after 20 mini the reaction is 19 ‘minutes then what amount ui 30 minutes? Solution. As tvs = 10 minutes = eos 40 0h patties? | at Hence amount left after 20 minutes FO) Scanned with CamScanner sant added after 20 minutes amount after 20 minutes =10 ¢ +20 g=g9 =305 otal jotbe ext 10 minutes the amount will be conta left after 30 minutes rere -smountlef 99 - 18 total amount 60 *100= 25%, action 2NO(8) + Cle) —— > 2NOCI g) I end ie lowing dat roti ad B Tnitial Rate of ntration® Formation of one) NOC! (mol [-1 min) No} (Cll 100.10 0.18 0.10 (0.20 0.36 0.20 0.20 1.44 ‘pune order ofreaction with respect to NO and Cc) 20x10 (25 x10}? (30105) = 1.07% 10° 1? mot? 5 log 42 = __Fa hy” Ba0xa4 | TT | or tog 207x109 [ 269%10° 2903x8518 e E, = 5.58% 108) B= 58.3 kd (iv) Calculation of A he= Ae Po/BT E, 303K log k= lox A- 553.10" log A=log (2.67% 108)— pao F308 x 814 xBO0 8.0565 ouhatist to Cli 21.148 * 104. Rewer ‘she numerical value of the rate constant at * ee ct . . Consider the following data for the reaction ern. () 2nd order in NO and 1st order in Cy (#9 180 A+B» Products Shue! came . _ Run Tnitial Cone. | Initial Rate [Bh conser the data forthe reatio between A and B Are BB) Tnitial Rate 1 01M 10M 21x10" (mol I=) (mol Lt ) 2 02M 10M 34x10% 7 300K | At d20K 3 0.2M 20M 84x 10% Determine overall ‘order, order with respect to A and ia eo Oe oe EES that with respect to B. x104 | 60x10% | 47 _ Solution. Rate =k (AJP(B]*, Overall order = 2+ Order wrt 1ox103 | 60x10°5| 16x10?) — ‘A= 2, order w.r.t, B= 0. Caleate For the reaction, A+B —» produets, the favour order wrt, Aand B Bree rates were obtained at various £16? initia (i) rate constant at 300 concentrations (i) the energy of activation EN, | Al mot tt | (BI mot Geode ace (je) the pre-exponential factor. (mol L715) ion () From the given data, the rate law comes out to ; aa Ol 0.05 Re 2 02 ol 0.10 =, (APB) Order w.r.t. A=2 3 0.1 O2 0.05 - Order w.r.t. B Write sherate award find the rate constant ‘of the above Now oa reaction. = MAPIBP’ (aPiB] eetution. Suppose rate = MALET 0 50104 rreaseL, 0.05 0-110 oe = _ (0.1 . @axiote (x10 for case, 0:10 eae wD = 2,67 x 108 L? mol? s* for case TT, 0.08 = MOAN —— Scanned with CamScanner 224 Comprehensiue eT, From equation (ii) and (i) we get Qt or 2=2x or x=1 0.05 © Similarly from equation (iii) and (i we get, 998 = ay or 120" or 0.05 log 1 = y log 2 0=yx0.3010 or y=0 ‘The rate law expression is, r= A(AJ[B)?= KIA] The value of =0.5 5. _— ial Fora first order reaction, show that time required for completion of 99.9% of reaction is 3 times the time required for completion of 90% of the reaction. oR Show that, ty 4, Styqy fora first order reaction. Solution. For a first order reaction, we know that and wid) Now divide () by (i) toon _ 3x 2303 to0% "2308 too. = Boor For a certain reaction, it takes § minutes for the initial ‘concentration of 0.5 mole per litre to become 0.25 mole per litre ed cn fi minutes to become 0.125 mole per lite. What is order of this reaction and why is it so? Calculate constant for the reaction. for Clea the rae Solution. For first order reaction, rate constant (b) is given by: From given data, a= 0.5 mol L-1, after ¢=5 minutes, aaa .25 mol L- 303 2.303 x 0.3010 Bmin 87=~— Smin=02986 my, During next 5 minutes, total time=19 ,: a= 0.5 mol L* (a= x) = 0.125 mol L-1 2.303 05 ak 10 min oe 0.125 2.303 log 4 = 2.303 x 0.6020. = Jo min 10min = 91386 m:,, ‘ ‘The following rate data was obtained for thy thermal decomposition of SO,Cl.{a) at a constany 2NO.La) ‘The rate law for this reaction is rate = k{NO}"(0,)- Propose a mechanism for the above reaction. Solution. The rate law indicate that order of reaction is 2 wart. NO and 1 w.r-t, O,. The possible mechanism for given reaction may be, NO +0, ee NO, (fast step) Scanned with CamScanner ee, Concho eH Rate constant k of @ reaction varies with, according to the equation: ing. x, NO,+NO—sNO,+NO, —_..(slow step) Overall reaction, (by addition of two steps) 2NO +0,—> 2NO, As slowest step of mechanism of reaction determine there E, is the eneray of activation forthe rating, the rate of reaction, graph is plotted for log k versus 4 straight tine wig, s rate = lt, [NO,] [NO] = 6670 K is obtained. Calculate energy of activatg But —_[NO,] =k, [NO][0,} reaction. State the units. (R= 8.314 JK mot) "a, NO, san intermediate specie, and Solution. "The given equation for the variation otra, its formation is in equilibrium state) _ with temperature is an equation of straight lie rate = h, hy [NO][O,][NO] intercept form, ie., = mx +e where, slope ay = R{NO}*[0,] m=-—Ea (where k is rate constant and k = k,l.) 2.303 RR ‘The above expression of rate law derived from proposed But slope is given to be ~ 6670 K. ‘mechanism is same as in given data, Bo pre EEE Tie decomposition of a compound is found to follow a 2303 R x 6670 _ 2303 8314.6 first-order rate law. If it takes 15 minutes for 20 percent of 1 1 original material to react, calculate (i) the specific rate constant, = 12771144 J mol = 1.28 x 108 Jn (i thetimeat which 10percent ofthe original material remains wom rye decomposition of Clj0, at 400 Kin th unreacted. 0; in the gosply to Cl, and O, is Ist order reaction. Solution. (i Specific rate constant, k for first order reaction (@ After 55 seconde at 400K, the presuref ja eee from 0.062 to 0.044 atm. Calculate the rate constant. (i Calculate the pressure of ClO, after 100s decomposition at this temperature. 2803 a _ 220, t Be a-x = 6.24 x 10° sec _ 100 303 ter 1D0=0\5 915 O08 mit Solution. = 0.0148 min 1.48 * 10% min (i) When 10% of original reactant remains unreacted, oe 00 see 90% of reaction is complete. We are required to calculate t, 3 = 2303 ,,, 062 of reaction. i 824% 108s 09 8 ax 2303, 100 2.303, 8 log ———_ = ——— 0.62 3. x100 _ 9 97 98 309-90 * Goran * Joe 10 or log O82. — S24 x10 2100 - oat 55.6 minutes. or (a~x) = 0.033 atm. Scanned with CamScanner r ical kinetics is the study of chemical renetions fr Tenet to reaction rates. It also deals with the rs eontrolling the rate of a reaction and by which Reaction rates can be altered. The kinetic data is to predict the mechanism of a reaction, ment of atoms and formation of intermediates, ‘turrate of a renction is concerned with decrease in e tration of reactants or increase in the ration of products per unit time. It can be as instantaneous rate at a particular instant rune and average rate over a large interval of time. Fora reaction , RP, the rato of reaction is: foRJarAED at At ‘A number of factors such as nature of reactants, ‘onentration of reactants, temperature, presence of @ ‘otalys,etc., affect the rate of a reaction. concent '» Rate Expression and Rate constant Mathematically representation of a reaction rate is given Ipnaelaw. It is determined experimentally and cannot bepredicted. Forthe reaction of the type, Aa + bB——> cC + aD Rate « [A}* [BP : aR _ app Diferentil rate equation, - > = # (AV (BP ‘there, is called rate constant. It is the proportionality factor in the rate law. Rate constant and order of a feaction can be determined from rate law or its ‘otegrated rate equation. "Order of a reaction with respect to a reactant is the Power ofits concentration which appears in the rate law ‘uation, The order of a reaction is the sum of all such Faw ofconcentration of terms for different reactants eee Tate expression, rate = i [A}* [B]"; X is order tA, Yis order w.r.t. B. ofa reaction can be 0, 1, 2 oF fractional. ‘nits of rate constant, k, are: rate _Cone., 1 IAF [BP time (Cone) “Msn ore of rosin Summary Psuedo first-order ler reactions are such reactions whic ‘pponr to be of high order but fllow low order oy fre order kinetics, For example, CH,COOC,H, + 1,0 —!, cH,coon + ¢,H,018 rate = k [CH,COOC,H,] [11,0] As (H,0] is constant, therefore rate =f’ [CH,COOC.H,}; ere ¥ (H,0]=h) Molecularity is defined only for an elementary reaction. Its values are limited from 1 to 3 whereas order can be 0, 1, 2,3 or even a fraction. Molecularity eannot be 2er0 or a non-integer. Molecularity and order of an elementary reaction are same. Many reactions occur as a sequence of elementary steps that make up the overall reaction mechanism. Integrated rate equations: ( Zero-order reactions: RP Ro-R, Concentration °o1R—> Tine (i First-order reactions: RP In (A vs tplot Scanned with CamScanner 228 For a typical first-order gas phase reaction, Ag— By + Cy if p; = initial pressure of A and Px * Pp * Pe total pressure at time ¢ then, 2303, py k= =~ log PL PA © Halflife of a reaction: (® Fora zero-order reaction: fy (ii For a first-order reaction: ty That is, t,, is independent of [R]o. © Temperature dependence of rate constants is described by Arrhenius equation A rise in temperature increases the rate of a reaction for two reasons: the total number of collisions possible per unit time is increased slightly; and, more importantly, the fraction of collisions that are sufficiently energetic to be effective is increased dramatically. ‘The Arrhenius equation is, k= Ae“Eo/RT or Ink=— 2a RT (Here, k = rate constant, E, = activation energy, R= gas constant, A = frequency or pre exponential factor). If h, and k, are the values of rate constants at temperatures T, and T, respectively, then vos Ba ‘| hy -2303R(T, 7 |’ +nA or log 2 = _Fa_[E=-T iy 2303R[ TH Intercept = In A t Slope = =-EyR 2 i is Ink vs 4 plot Comprchensing The equation clearly shows thay Sy temperature arlowering of, wi ptt tag, in the rate of reaction. toagity A catalyst increases the rate of " providing an alternative pathway’ "ey activation energy, to the same prow auch larger fraction of collisions apes the changed reaction mechanigm «lly! ; : anism. Cat involved in the reaction mechanisn, ae but are regenerated before the reacties" ht ot we ray | }eacton path without catalyst =. re |e 3 Energy off [Ss 2 |Reactants // seta | ee 5 "Reaction pa\ "in, | |e 3 with catalyst \Calyst | @ Prot Reaction coordante Collision theory of chemical reactions: A mags occurs due to collisions. Particles require niniws activation energy and correct alignment foreaisia| to be effective. The collisions must provide sufi energy to cause the breaking and forming of ti producing new particles. The number of collisions per second per unit whiz of a reaction is known as collision frequency (3 For a reaction, A+ B——> Products, RT rate = zap. ap = collision frequency and e~®+/®? = ‘exit molecules with energy equal to or greater than, is also called Boltzmann factor.) b To account for effective collisions another feo called probability or steric factor is into! Accordingly, -z,/Rt, rate constant, k= P. zn -€ Scanned with CamScanner

You might also like